SSC CGL Tier-I 03 March 2020 Shift-II Previous Year Paper

SSC CGL

(3 March 2020 Shift-II)

General Intelligence and Reasoning 

Q. 1 In a certain code language, ‘HAMMER’is written as ‘ICPQJX’. How will ‘WRENCH’ be written as in that language? 

A XTIRIN 

B XTIRHN 

C XTHRIN 

D XTHRHN 

Answer: D 

 

Q. 2 Select the option figure in which the given figure is embedded (rotation is not allowed). 

A

B

C

D

Answer: A 

Q. 3 In certain code language, ‘PEN’ is coded as ‘321028’. How will ‘TUB’ be coded as in that language? 

A 40422 

B 40424 

C 42404 

D 44024 

Answer: B 

 

Q. 4 Select the option that depicts how the given transparent sheet of paper would appear if it is folded at the dotted line.

A

B

C

D

Answer: C 

Q. 5 Select the number that can replace the question mark (?) in the following series. 

55, 58, 64, ? , 85 

A 77 

B 70 

C 67 

D 73 

Answer: D 

 

Q. 6 Select the letter that can replace the question mark (7?) in the following series. 

J, M, P, ?, V, Y 

A T 

B O 

C S 

D R 

Answer: C 

 

Q. 7 In the given Venn diagram, the ‘circle’ represents ‘ladies’, the ‘triangle’ represents ‘teachers’, and the ‘rectangle’ represents ‘unmarried persons’. The numbers given in the diagram represent the number of persons in that particular category. 

How many married ladies are teachers? 

A 9 

B 11 

C 3 

D 6 

Answer: C 

 

Q. 8 Four number-pairs have been given, out of which three are alike in some manner and one is different. Select the number-pair that is different from the rest. 

A 39 – 72 

B 28 – 60 

C 42 – 12 

D 57 – 38 

Answer: D 

 

Q. 9 Select the option in which the words share the same relationship as that shared by the given pair of words. Faculty : Teachers 

A Ants : Flock 

B Fleet : Trucks 

C Galaxy : Apartments 

D Colony : Wolves 

Answer: B 

 

Q. 10 Select the figure that can replace the question mark (?) in the following series. 

Answer: D 

 

Q. 11 Three different positions of the same dice are shown. Select the symbol that will be on the face opposite to the one showing <’. 

A # 

B @ 

C & 

D $ 

Answer: B 

 

Q. 12 Read the given statements and conclusions carefully. Assuming that the information given in the statements is true, even if it appears to be at variance with commonly known facts, decide which of the given conclusions logically follow(s) from the statements. Statements: 

1. All dogs are lions. 

2. No elephant is a lion. 

Conclusions: 

I. No dog is an elephant. 

II. No lion is a dog. 

III. Some elephants are dogs. 

A Only conclusion I follows. 

B Only conclusions II and ITI follow. 

C Only conclusions I and III follow. 

D Only conclusionII follows. 

Answer: A 

 

Q. 13 Eight words have been given, out of which seven are alike in some manner and one is different. Select the odd word. Monitor, Headphone, Mouse, Keyboard, Windows, Printer, Scanner, Speaker 

A Mouse 

B Printer 

C Monitor 

D Windows 

Answer: D 

 

Q. 14 Select the option that is related to the third number in the same way as the second number is related to the first number. 2809 : 53 :: 1524 : ? 

A 31 

B 35 

C 32 

D 33 

Answer: C 

 

Q. 15 Four letter-clusters have been given, out of which three are alike in some manner and one is different. Select the odd letter-cluster. 

A JMPS 

B BHEK 

C ADGJ 

D FHKM 

Answer: D 

 

Q. 16 In an exam of 80 questions, a correct answer is given +1 mark, a wrong answer is given —1 mark, and if a question is not attempted there are zero marks. If a student attempted only 80% of the questions and got 32 marks, then how many questions did he answer correctly? 

A 32 

B 48 

C 16 

D 56 

Answer: B 

 

Q. 17 Select the correct mirror image of the given figure when a mirror is placed on the right of the figure.

Answer: A 

 

Q. 18 Study the given pattern carefully and select the number that can replace the question mark (?) in it. 

A 212 

B 209 

C 261 

D 122 

Answer: C 

 

Q. 19 Arrange the following words in a logical and meaningful order. 

1. Hexagon 

2. Nonagon 

3. Pentagon 

4. Heptagon 

5. Octagon 

A 3-1-4-5-2 

B 1-4-5-2-3 

C 1-3-4-5-2 

D 4-3-1-2-5 

Answer: A 

 

Q. 20 Select the letter-cluster that can replace the question mark (?) in the following series. BRH, ZUD, ? , VAV, TDR, RGN 

A XZZ 

B XYZ 

C XZY 

D XXZ 

Answer: D 

 

Q. 21 A + B means ‘B is the brother of A’: 

A B means ‘A is the mother of B’: 

A × B means ‘A is the father of B’: 

A ÷ B means ‘A is the son of B’. 

If P + R × T Q ÷ S + U, then how is S related to R? 

A Grandfather 

B Brother 

C Grandson 

D Son-in-law 

Answer: D 

 

Q. 22 Study the given pattern carefully and select the number that can replace the question mark (?) in it. 

A 23 

B 24 

C 22 

D 21 

Answer: C 

 

Q. 23 Which two signs should be interchanged to make the given equation correct? 

36 ÷ 2 × 12 + 3 − 6 = 24 

A − and ÷

B × and

C + and ×

D ÷ and ×

Answer: B 

 

Q. 24 ‘Cardiologist’ is related to ‘Heart’ in the same way as ‘Neurologist’ is related to ‘_______’. 

A Lungs 

B Brain 

C Teeth 

D Ears 

Answer: B 

 

Q. 25 How many triangles are there in the given figure? 

A 8 

B 10 

C 12 

D 6 

Answer: B 

 

General Awareness 

Q. 26 The bodies that glow during the night sky are known as: 

A Celestial bodies 

B Asteroids 

C Constellation 

D Meteroid 

Answer: A 

 

Q. 27 Archaeologist R Nagaswamy was honoured at the Silver Jubilee International Conference of Art by which country? 

A Bhutan 

B Nepal 

C Bangladesh 

D China 

Answer: C 

 

Q. 28 In which state has the Khadi and Village Industries Commission (KVIC) opened the first silk processing plant? 

A Maharashtra 

B Tamil Nadu 

C Karnataka 

D Gujarat 

Answer: D 

 

Q. 29 In January 2020, Home Minister Amit Shah released a book ‘Karmayoddha Granth’. This book is based on the life of ________. 

A Jawaharlal Nehru 

B Narendra Modi 

C Sardar Vallabhbhai Patel 

D Mahatma Gandhi 

Answer: B 

 

Q. 30 Which state’s Legislative Assembly adopted a new logo consisting of the national emblem and foxtail orchid (Rhynchostylis Retusa), the state flower, in January 2020? 

A Mizoram 

B Meghalaya 

C Tripura 

D Arunachal Pradesh 

Answer: D 

 

Q. 31 In which of the following states is the Madhavpur Mela celebrated? 

A Gujarat 

B Madhya Pradesh 

C Uttar Pradesh 

D Bihar 

Answer: A 

 

Q. 32 Which of the following festivals means “merry marking of the Gods”? 

A Diwali 

B Pongal 

C Lai Haraoba 

D Makar Sankranti 

Answer: C 

 

Q. 33 Ajatashatru, a ruler of the Haryanka Dynasty, was the son of _____. 

A Naga-Dasak 

B Udayin 

C Anurudha 

D Bimbisara 

Answer: D 

 

Q. 34 Which of the following glands is present between the lungs? 

A Thymus 

B Pituitary 

C Hypothalamus 

D Pineal 

Answer: A 

 

Q. 35 Which of the following is not a vertebrate? 

A Snail 

B Bird 

C Mammal 

D Fish 

Answer: A 

 

Q. 36 Which law of physics states that the force between the two electric charges reduces to a quarter of its former value when the distance between them is doubled? 

A Coulomb’s Law 

B Stefan’s Law 

C Pascal’s Law 

D Hooke’s Law 

Answer: A 

 

Q. 37 In which of the following countries was the 95th edition of the prestigious Hastings International Chess Congress held? 

A France 

B Australia 

C England 

D Belgium 

Answer: C 

 

Q. 38 When we cut an onion, the synthase enzyme converts the amino acid sulfoxides of the onion into which acid? 

A Sulfenic acid 

B Citric acid 

C Nitric acid 

D Sulphuric acid 

Answer: A 

 

Q. 39 Which of the following is the major component of vinegar? 

A Citric acid 

B Lactic acid 

C Nitric acid 

D Acetic acid 

Answer: D 

 

Q. 40 Which of the following was previously known as ‘the Lady Willingdon Park’? 

A Buddha Jayanti Park 

B Mughal Gardens 

C Deer Park 

D Lodhi Gardens 

Answer: D 

 

Q. 41 ‘Industry 4.0’ is a complex cyber-physical system which synergies production with digital technologies. The Ministry of Railways and the Department of Science and Technology have joined hands in partnership with which institution for taking up a unique project on ‘Industry 4.0’? 

A IIT Kanpur 

B IIT Bombay 

C IIT Madras 

D IIT Delhi 

Answer: A 

 

Q. 42 Who among the following was honoured with the 50th Dadasaheb Phalke Award? 

A Anupam Kher 

B Naseeruddin Shah 

C Amitabh Bachchan 

D Kamal Haasan 

Answer: C 

 

Q. 43 In which district of Karnataka is the Brahmagiri Wildlife Sanctuary located? 

A Hassan 

B Mandya 

C Udupi 

D Kodagu 

Answer: D 

 

Q. 44 Who among the following was the last ruler of the Nanda dynasty? 

A Dhanananda 

B Panduka 

C Govishanaka 

D Kaivarta 

Answer: A 

 

Q. 45 What was India’s position in the Brand Finance Nation ranking of 2019? 

A Seventh 

B Sixth 

C Third 

D Fifth 

Answer: A 

 

Q. 46 Who became the first Indian equestrian to qualify for the Tokyo Olympics 2020? 

A Fouaad Mirza 

B Amit Sinsinwar 

C Sehej Singh Virk 

D Amar Sarin 

Answer: A 

 

Q. 47 In which state was the Global Investors Meet, ASCEND 2020 organised? 

A Maharashtra 

B Kerala 

C Rajasthan 

D Gujarat 

Answer: B 

 

Q. 48 Wings India 2020 is scheduled to be held in which of the following airports? 

A Begumpet Airport 

B Warangal Airport 

C Rajahmundry Airport 

D Vijayawada Airport 

Answer: A 

 

Q. 49 Borra caves are situated on the East Coast of India in which of the following hills? 

A Nallamala Hills 

B Horsley Hills 

C Nagari Hills 

D Ananthagiri Hill 

Answer: D 

 

Q. 50 In which year was the foundation stone for the Gateway of India laid in Bombay (now Mumbai)? 

A 1913 

B 1915 

C 1905 

D 1920 

Answer: A 

 

Quantitative Aptitude 

Q. 51 A can complete a certain piece of work in 40 days. B is 25% more efficient than A and C is 28% more efficient than B. They work together for 5 days. The remaining work will be completed by B alone, in: 

A 20¾ days 

B 16⅕ days 

C 16⅗ days 

D 20½ days 

Answer: C 

 

Q. 52 A race track is in the shape of a ring whose inner and outer circumferences are 440 m and 506 m,respectively. What is the cost of levelling the track at ₹6/m2 ? (Takeπ = 22/7

A ₹ 18,966 

B ₹ 24,832 

C ₹ 19,866 

D ₹ 29,799 

Answer: D 

 

Q. 53 The compound interest on a certain sum at 10% p.a. for 2⅓ years is ₹1,201.60, interest compounded yearly. The sum is: 

A ₹ 4,200 

B ₹ 4,800 

C ₹ 5,400 

D ₹ 4,500 

Answer: B 

 

Q. 54 The given table represents the number of engineers recruited by four companies A, B, C and D over the years. Study the table carefully and answer the question that follows. 

The ratio of the total number of engineers recruited by companies A and B in 2015 and 2018 to the total number of engineers recruited by C and D in 2014 and 2018,is: 

A 28 : 19 

B 13 : 21 

C 17 : 14 

D 9 : 14 

Answer: C 

 

Q. 55 Two bottles of the same capacity are 35% and 33⅓% full of orange juice, respectively. They are filled up completely with apple juice and then the contents of both bottles are emptied into another vessel. The percentage of apple juice in the mixture is: 

A 34⅙

B 64⅓

C 60⅔

D 65⅚

Answer: D 

 

Q. 56 In △ABC, AB = AC and AL is perpendicular to BC at L. In △DEF, DE = DF and DM is perpendicular to EF at M.If (area of △ABC)/(area of △DEF) = 9:25, then (DM+AL)/(DMAL) is equal to: 

A 6 

B 4 

C 3 

D 5 

Answer: B 

 

Q. 57 The given table represents the number of engineers recruited by four companies A, B, C and D over the years. Study the table carefully and answer the question that follows. 

The number of the years in which the number of engineers recruited by company D is less than the average number of engineers recruited by B in the given six years,is: 

A 3 

B 1 

C 4 

D 2 

Answer: A 

 

Q. 58 The value of (18 ÷ 2 of ¼) ×(⅔÷¾×⅝)÷(⅔÷¾ of ¾) is:

A 10 ⅔

B 8⅝

C 16 ⅞

D 2 ⁷/₆₄

Answer: C 

 

Q. 59 If 2 sin θ + 15 cos2 θ = 7, 0< θ < 90, then tan θ + cos θ + sec θ =

A 3⅗

B 3

C 3⅘

D 4

Answer: A 

 

Q. 60 △ABC is an equilateral triangle and AD⊥BC, where D lies on BC. If AD = 4√3cm, then what is the perimeter (in cm) of △ABC? 

A 27 

B 24 

C 30 

D 21 

Answer: B 

 

Q. 61 PQRS is a cyclic quadrilateral in which PQ = x cm, QR= 16.8 cm, RS = 14 cm, PS = 25.2 cm, and PR bisects QS. What is the value of x ? 

A 18 

B 21 

C 28 

D 24 

Answer: B 

 

Q. 62 If (sec θ−tan θ)/(sec θ+tan θ)=⅗, then the value of  (cosec θ+cot θ)/(cosec θ−cot θ) is:

A 24 +√15

B 31 +8√15

C 27 +√15

D 33 +4√15

Answer: B 

 

Q. 63 The given table represents the number of engineers recruited by four companies A, B, C and D over the years. Study the table carefully and answer the question that follows. 

The total number of engineers recruited by company A in 2014 to 2017 is what percentage more than the total number of engineers recruited by all four companies in 2019? 

A 3 

B 3.5 

C 2.5 

D 4 

Answer: D 

 

Q. 64 If x is the mean proportional between 12.8 and 64.8 and y is the third proportional to 38.4 and 57.6, then 2x : y is equal to: 

A 2 : 3 

B 4 : 5 

C 3 : 4 

D 1 : 2 

Answer: A 

 

Q. 65 The average of the first four numbers is three times the fifth number. If the average of all the five numbers is 85.8, then the fifth number is: 

A 33 

B 34 

C 39 

D 29 

Answer: A 

 

Q. 66 Quadrilateral ABCD circumscribes circle. If AB = 8 cm, BC = 7 cm and CD = 6 cm,then the length of AD is: 

A 6 cm 

B 7.5 cm 

C 7cm 

D 6.8 cm 

Answer: A 

 

Q. 67 The expression is (a + b c)3 + (a b + c)3 − 8a3 equal to:

A 6a(a b + c)(c a b)

B 3a(a + b c)(a b + c)

C 6a(a + b c)(a b + c)

D 3a(a b + c)(c a b)

Answer: A 

 

Q. 68 If x4 + x2y2 + y4 = 21 and x2 + xy + y2 = 7, then the value of (x½ + y½) is:

A ⁵/₂

B ⁷/₄

C ⁵/₄

D ⁷/₃

Answer: C 

 

Q. 69 The given table represents the number of engineers recruited by four companies A, B, C and D over the years. Study the table carefully and answer the question that follows. 

The total number of engineers recruited by company B in 2014 and 2017 is what percentage of the total number of engineers recruited by C during 2015 to 2019? 

A 38.4 

B 38.2 

C 39.2 

D 37.8 

Answer: C 

 

Q. 70 The value of the expression cosec(85+ θ) − sec(5θ) − tan(55+ θ) + cot(35θ) is: 

A ³/₂

B 0 

C -1 

D 1 

Answer: B 

 

Q. 71 When 200 is divided by a positive integer x , the remainder is 8. How many values of x are there? 

A 7 

B 5 

C 8 

D 6 

Answer: C 

 

Q. 72 If x2 + 3x + 1 = 0, then what is the value of x6 + x

A 324 

B 322 

C 318 

D 327 

Answer: B 

 

Q. 73 Anu fixes the selling price of an article at 25% above its cost of production. If the cost of production goes up by 20% and she raises the selling price by 10%, then her percentage profit is (correct to one decimal place): 

A 16.4% 

B 14.6% 

C 13.8% 

D 15.2% 

Answer: B 

 

Q. 74 A and B start moving towards each other from places X and Y, respectively, at the same time. The speed of A is 20% more than that of B. After meeting on the way, A and B take 2½hours and x hours, now to reach Y and X, respectively. What is the value of x ? 

A 3⅗

B 3⅔

C 3½

D 3⅖

Answer: A 

 

Q. 75 A dealer marks an article 40% above the cost price and sells it to a customer, allowing two successive discounts of 20% and 25% on the marked price. If he suffers a loss of ₹ 140, then the cost price (in ₹) of the article is: 

A 900 

B 840 

C 872 

D 875 

Answer: D 

 

English Comprehension 

Q. 76 In the sentence identify the segment which contains the grammatical error. 

The modern man is busy acquiring more and more wealth and designing ways to invest it in more sense pleasures. 

A designing ways to invest it 

B in more sense pleasures 

C modern man is busy 

D acquiring more and more wealth 

Answer: B 

 

Q. 77 Select the most appropriate word to substitute the underlined word of the given sentence. If no substitution is required, select ‘No improvement’. 

There is a great degrade in values in modern age. 

A degradation of values 

B deliberation for values 

C demonstration from values 

D No improvement 

Answer: A 

 

Q. 78 Select the most appropriate word to fill in the blank. 

It is an ______ day to start your new business. 

A audacious 

B ominous 

C auspicious 

D occasional 

Answer: C 

 

Q. 79 Select the synonym of the given word. 

PATHETIC 

A Pitiful 

B Dull 

C Insignificant 

D Curious 

Answer: A 

 

Q. 80 Select the antonym of the given word. 

HILARIOUS 

A Blithe 

B Merry 

C Sad 

D Happy 

Answer: C 

 

Q. 81 Select the correct indirect form of the given sentence. 

He said to me, “What are you doing?” 

A He said what I had been doing. 

B He said that what I was doing 

C He asked me what I was doing 

D He asked me that what was I doing 

Answer: C 

 

Instructions 

In the following passage, some words have been deleted. Fill in the blanks with the help of the alternatives given. Select the most appropriate option for each blank. 

Comprehension: 

Machines were made to be man’s servants. Yet, man has grown so (1)______ on them that they are in a fair way to become his (2)______. Already men spend most of their lives looking after and waiting (3)______ machines. Machines are very stern bosses. They must be fed with coal and (4)______ petrol to drink and oil to wash with and must be kept at an (5)______ temperature. 

Q. 82 Select the most appropriate option for blank no. 1 

A subordinate 

B dependent 

C inferior 

D helpless 

Answer: B 

 

Q. 83 Select the most appropriate option for blank no. 2 

A masters 

B victims 

C slaves 

D administrators 

Answer: A 

 

Q. 84 Select the most appropriate option for blank no. 3 

A from 

B upon 

C under 

D into 

Answer: B 

 

Q. 85 Select the most appropriate option for blank no. 4 

A given 

B gives 

C gave 

D give 

Answer: A 

 

Q. 86 Select the most appropriate option for blank no. 5 

A outdated 

B optimum 

C optional 

D optimist 

Answer: B 

 

Q. 87 Given below are four jumbled sentences. Out of the given options pick the one that gives their correct order. 

A. Can I borrow your camera? 

B. I will give it back to you next week. 

C. I am going to jungle safari tomorrow. 

D. My friend told me that jungle is beautiful in these days 

A CDAB 

B BACD 

C ADBC 

D CADB 

Answer: A 

 

Q. 88 Select one word for the following group of words. 

One who leaves his own country to settle in another 

A Foreigner 

B Emigrant 

C Tourist 

D Native 

Answer: B 

 

Q. 89 Select the wrongly spelt word. 

A Practicle 

B Flexible 

C Flashy 

D Elegant 

Answer: A 

 

Q. 90 In the sentence identify the segment which contains the grammatical error. 

The Prime Minister, along with the other ministers have left for America. 

A for America 

B The Prime Minister along with 

C have left 

D the other ministers 

Answer: C 

 

Q. 91 Select the wrongly spelt word. 

A Ostentasion 

B Mansion 

C Extension 

D Persuasion 

Answer: A 

 

Q. 92 Select the antonym of the given word. 

VICIOUS 

A Baneful 

B Unfortunate 

C Virtuous 

D Sinful 

Answer: C 

 

Q. 93 Select the most appropriate word to fill in the blank. 

He ______ a heinous crime. 

A happened 

B committed 

C made 

D occurred 

Answer: B 

 

Q. 94 Select the most appropriate meaning of the given idiom 

On shank’s mare 

A On an elephant 

B On a lion 

C On a bicycle 

D On foot 

Answer: D 

 

Q. 95 Select the most appropriate meaning of the given idiom 

A snake in the grass 

A A well-wisher 

B Difficult to find 

C A good friend 

D A secret enemy 

Answer: D 

 

Q. 96 Select one word for the following group of words. 

A period of ten years 

A Fortnight 

B Millennium 

C Decade 

D Century 

Answer: C 

 

Q. 97 Select the synonym of the given word. 

PENITENCE 

A Patience 

B Repentance 

C Misery 

D Admiration 

Answer: B 

 

Q. 98 Given below are four jumbled sentences. Out of the given options pick the one that gives their correct order. 

A. Nevertheless, sound health, economic security and mental satisfaction are desired by all 

B. A change that is conducive to happiness may be termed as progress. 

C. But different people find happiness in different things. 

D. So, If a change contributes to the growth of these factors, it is progress. 

A BDCA 

B ABCD 

C BCAD 

D DBCA 

Answer: C 

 

Q. 99 Select the correct active form of the given sentence. 

The thief was being arrested by the police. 

A The police has arrested the thief 

B The police arrested the thief. 

C The police had arrested the thief. 

D The police was arresting the thief. 

Answer: D 

 

Q. 100 Select the most appropriate segment to substitute the underlined segment of the given sentence. If no substitution is required select ‘No improvement’ 

A man in need pleaded for help . 

A promised for help 

B commanded to help 

C No improvement 

D requested for helping 

Answer: C 

SSC CGL Tier-I 03 March 2020 Shift-I Previous Year Paper

SSC CGL

(3 March 2020 Shift-I)

General Intelligence and Reasoning 

Q. 1 Which two numbers should be interchanged to make the given equation correct? 

9 + 7 × 5 − 18 ÷ 2 = 3 × 4 − 10 + 45 ÷ 5 

A 18 and 45 

B 7 and 4 

C 2 and 5 

D 9 and 3 

Answer: B 

 

Q. 2 Arrange the following words in the order in which they appear in an English dictionary. 

1. Rightly 

2. Rigidly 

3. Righteous 

4. Rigour 

5. Rights 

A 3, 5, 1, 4, 2 

B 3, 1, 5, 4, 2 

C 3, 1, 5, 2, 4 

D 1, 3, 5, 2, 4 

Answer: C 

 

Q. 3 Which of the option figures is the exact mirror image of the given figure when the mirror is held at the right side?

A

B

C

D

Answer: D 

 

Q. 4 The ratio of the present ages of Asha and Lata is 5 : 6. If the difference between their ages is 6 years, then what will be Lata’s age will be after 5 years? 

A 35 

B 41 

C 45 

D 50 

Answer: B 

 

Q. 5 Select the option in which the numbers are related in the same way as are the numbers in the given set. (269, 278, 296) 

A (313, 322, 340) 

B (577, 586, 598) 

C (419, 430, 448) 

D (109, 118, 128) 

Answer: A 

 

Q. 6 Four letter-clusters have been given, out of which three are alike in some manner, while one is different. Select the odd letter-cluster. 

A BYGT 

B NMSH 

C FVKO 

D DWIR 

Answer: C 

 

Q. 7 Amit is the brother of Sonia. Jyoti is the sister ofNikita. Sonia is the daughter of Satish’s father. Nikita is the daughter of Kavinder. Jyoti is the mother ofAmit. Mukeshis Nikita’s only sister’s husband. Howis Satish related to Kavinder? 

A Son 

B Brother 

C Grandson 

D Son-in-law 

Answer: C 

 

Q. 8 Four words have been given, out of which three are alike in some manner, while one is different. Select the odd word. 

A Ferry 

B Yacht 

C Submarine 

D Ship 

Answer: C 

 

Q. 9 In a certain code language, WARDROBE is written as YXVYXHIJV. How will ACCURATE be written as in that language? 

A BZHPXTBV 

B CZHPYTBV 

C CZGPXTBV 

D DZGPXTBV 

Answer: C 

 

Q. 10 Select the number that can replace the question mark (?) in the following series. 

17, 21, 30, 46, 71, ? 

A 105 

B 96 

C 107 

D 101 

Answer: C 

 

Q. 11 The given Venn diagram represents employees in an organisation: 

Thetriangle represents executives, the circle represents females, the rectangle represents MBAs and the square represents technical staff. The numbers given in the diagram represent the number of persons in that particular category. 

How many female executives are there in the organisation? 

A 10 

B 5 

C 15 

D 11 

Answer: B 

 

Q. 12 Read the given statements and conclusions carefully. Assuming that the information given in the statements is true, even if it appears to be at variance with commonly known facts, decide which of the given conclusions logically follow(s) from the statements. Statements: 

1. Some animals are elephants. 

2. Some elephants are tigers. 

Conclusions: 

I. Some animals are tigers. 

II. No tiger is an animal. 

A Only conclusion I follow. 

B Either conclusion I or II follows. 

C Neither conclusion I nor II follows. 

D Only conclusionII follows. 

Answer: B 

 

Q. 13 Two positions of the same dice are shown. Select the number that will be on the face opposite to the one showing 6.

A 4 

B 5 

C 1 

D 3 

Answer: C 

 

Q. 14 Select the set of letters that when sequentially placed in the blanks of the given letter series will complete the series. k_Imml_mk_mmk_Ikkl_m 

A k, l, m, k, k 

B l, k, m, k, k

C k, l, k, l, m 

D k, m, m, k, l 

Answer: C 

 

Q. 15 The sequence of folding a piece of paper and the manner in which the folded paper has been cut is shown in the following figures. How would this paper look when unfolded? 

A

B

C

D

Answer: B 

 

Q. 16 Select the option that is related to the third word in the same way as the second word is related to the first word. Medicine : Disease :: Food : ? 

A Energy 

B Nutrition 

C Thirst 

D Hunger 

Answer: D 

 

Q. 17 Select the letter-cluster that can replace the question mark (?) in the following series. 

CXB, HUI, MRP, ROW, ? 

A VKD 

B VKC 

C WLZ 

D WLD 

Answer: D 

 

Q. 18 In a certain code language, ‘HARVEST’ is coded as ‘22-21-7-24-20-3-10°. How will ‘FARMER’ be coded as in that language? 

A 20-7-14-21-3-8 

B 19-7-15-19-3-8 

C 19-7-15-20-3-7 

D 20-7-15-20-3-8 

Answer: D 

 

Q. 19 Select the option that is related to the third number in the same way as the second number is related to the first number and the sixth number is related to the fifth number. 

12 : 72 :: 18 : ? :: 22 : 242 

A 162 

B 164 

C 140 

D 160 

Answer: A 

 

Q. 20 Select the option in which the words share the same relationship as that shared by the given pair of words. Clock : Time 

A Ammeter : Current 

B Taseometer : Wind 

C Balance : Scale 

D Anemometer : Strains 

Answer: A 

 

Q. 21 Study the given pattern carefully and select the number that can replace the question mark (?) in it.

A 98 

B 84 

C 91 

D 78 

Answer: C 

 

Q. 22 How many rectangles are there in the given figure? 

A 34 

B 32 

C 30 

D 33 

Answer: D 

 

Q. 23 Select the option figure in which the given figure is embedded(rotation is NOT allowed).

Answer: A 

 

Q. 24 In the following equations, if ‘+’ is interchanged with ‘—’ and ‘6’ is interchanged with ‘7’, then which equation would be correct? 

A 67 – 76 + 43 = 100 

B 76 – 75 + 77 = 56 

C 62 – 67 + 76 = 83 

D 78 – 68 + 66 = 59 

Answer: A 

 

Q. 25 Select the figure that can replace the question mark (?) in the following series. 

Answer: A 

 

General Awareness 

Q. 26 Red worms have a structure named ______ which helps them in grinding their food. 

A Esophagus 

B Intestine 

C Crop 

D Gizzard 

Answer: D 

 

Q. 27 Prolific Indian painter Maqbool Fida Husain predominantly used which of these animals to depict a lively and free spirit in his paintings? 

A Elephants 

B Horses 

C Cows 

D Tigers 

Answer: B 

 

Q. 28 Sultan Qaboos bin Said of ________, the Arab world’s longest-serving ruler and with a reputation for quiet diplomacy passed away recently (2020). 

A Abu Dhabi 

B Oman 

C Dubai 

D Kuwait 

Answer: B 

 

Q. 29 Which of these institutions fixes the Repo Rate and the Reverse Repo Rate in India? 

A Comptroller and Auditor General of India 

B Reserve Bank of India 

C Ministry of Finance 

D State Bank of India 

Answer: B 

 

Q. 30 Sir Thomas Roe came as an official ambassador from King James I of England to which Mughal emperor’s court? 

A Akbar 

B Aurangzeb 

C Shah Jahan 

D Jahangir 

Answer: D 

 

Q. 31 What is the more common name for solid carbon dioxide? 

A Epsom 

B Dry Ice 

C Potash 

D Quicksilver 

Answer: B 

 

Q. 32 As of January 2020, Shri Bhupesh Baghel is the Chief Minister of which of the following states? 

A Odisha 

B Chhattisgarh 

C Jharkhand 

D Haryana 

Answer: B 

 

Q. 33 Name the media company that purchased the legendary studio of 21st Century Fox. 

A Time Warner 

B Viacom 

C Sony 

D Disney 

Answer: D 

 

Q. 34 Who among the following played the leading lady in the film ‘Mission Mangal’ that tells the dramatic true story of the women behind India’s first mission to Mars? 

A Deepika Padukone 

B Kareena Kapoor 

C Vidya Balan 

D Kajol 

Answer: C 

 

Q. 35 For which of the following sports was Dronavalli Harika, conferred with the prestigious Padma Shri award? 

A Chess 

B Badminton 

C Archery 

D Cricket 

Answer: A 

 

Q. 36 From India, who inaugurated the Kartarpur Corridor and flagged off the first set of pilgrims to the final resting place of Sikhism founder Guru Nanak Dev? 

A Narendra Modi 

B Amarinder Singh 

C Ram Nath Kovind 

D Manmohan Singh 

Answer: A 

 

Q. 37 Who is the first and currently the only batsman to score double hundreds in four consecutive test series? 

A A.B. de Villiers 

B Rohit Sharma 

C Virat Kohli 

D Brian Lara 

Answer: C 

 

Q. 38 The ruins of the ancient city of Hampi – capital of Vijayanagara – is located in which present day Indian state? 

A Haryana 

B Bihar 

C Karnataka 

D Telangana 

Answer: C 

 

 Q. 39 In which year Sanchi was discovered after being abandoned for nearly 600 Years? 

A 1820 

B 1818 

C 1814 

D 1816 

Answer: B 

 

Q. 40 Kolathunadu, Valluvanad and Thekkumkoor were ancient small-time kingdoms in which state of India? 

A Karnataka 

B Gujarat 

C Bihar 

D Kerala 

Answer: D 

 

Q. 41 What is the uniform GST rate that has been fixed up for lottery prizes by the GST Council? 

A 28% 

B 18% 

C 10% 

D 32% 

Answer: A 

 

Q. 42 As of February 2020, who is the President of Sri Lanka? 

A Gotabaya Rajapaksa 

B Chandrika Kumaratunga 

C Maithripala Sirisena 

D D.M. Jayaratne 

Answer: A 

 

Q. 43 Which of the following books is NOT written by Salman Rushdie? 

A An Area of Darkness 

B Shame 

C Midnight’s Children 

D The Satanic Verses 

Answer: A 

 

Q. 44 Which of these bones is NOT a part of the human ear? 

A Malleus 

B Stapes 

C Femur 

D Incus 

Answer: C 

 

Q. 45 The Araku Valley, a tourist resort, is located near which of these cities of South India? 

A Visakhapatnam 

B Madurai 

C Kochi 

D Mangalore 

Answer: A 

 

Q. 46 Veteran freedom fighter, social reformer and feminist Savithribai Phule hailed from which of the following states of India? 

A Odisha 

B Gujarat 

C Rajasthan 

D Maharashtra 

Answer: D 

 

Q. 47 Name the author who won the Sahitya Akademi Award 2019 for his book – An Era of Darkness: The British Empire in India. 

A Ramchandra Guha 

B Shashi Tharoor 

C Romila Thapar 

D Vikram Seth 

Answer: B 

 

Q. 48 Which of these words refers to the scientific study of domestic dogs? 

A Craniology 

B Chrematistics 

C Carpology 

D Cynology 

Answer: D 

 

Q. 49 Name the physicist who is credited with the discovery of the Neutron. This 1932 discovery led to his winning the Nobel Prize. 

A Enrico Fermi 

B J.S. Fleming 

C James Chadwick 

D Max Plank 

Answer: C 

 

Q. 50 The World Food Program (WFP) is the food assistance branch of the United Nations. Where is it headquartered? 

A Rome 

B New York 

C Paris 

D Brussels 

Answer: A 

 

Quantitative Aptitude

Q. 51 If the number 1005 x 4 is completely divisible by 8, then the smallest integer in place of x will be: 

A 0 

B 1 

C 4 

D 2 

Answer: A 

 

Q. 52 The given table shows the number (in thousands) of cars of five different models A, B, C, D and E produced during Years 2012-2017. Study the table and answer the questions that follow. 

If 2013 and 2014 are put together, which type of cars constitute exactly 25% of the total number of cars produced in those 2 years? 

A E 

B B 

C D 

D C 

Answer: C 

 

Q. 53 If x = 4 cosA + 5 sinA and y = 4 sin A + 5 cos A, then the value of x2 + y2 is:

A 25 

B 41 

C 0 

D 16 

Answer: B 

 

Q. 54 The radius of a circular garden is 42 m. The distance (in m) covered by running 8 rounds around it, is: (Take π = 22/7

A 2112 

B 1124 

C 3248 

D 4262 

Answer: A 

 

Q. 55 The given table shows the number (in thousands)of cars of five different models A, B. C, D and E produced during Years 2012-2017. Study the table and answer the questions that follow. 

The percentage decrease in the production of which type of car in 2017, with reference to 2016, was the maximum? 

A D 

B E 

C A 

D C 

Answer: C 

 

Q. 56 If x, y, z are three integers such that x + y = 8, y + z = 13 and z + x = 17, then the value of x2/yz is: 

A 7/5

B 18/11

C 1 

D 0 

Answer: B 

 

Q. 57 In △ABC, MN∥BC, the area of quadrilateral MBCN=130 sqcm. If AN : NC = 4 : 5, then the area of △ MAN is:

A 40 cm2

B 45 cm2

C 32 cm2

D 65 cm2

Answer: C 

 

Q. 58 A, B and can individually complete a piece of work in 24 days, 15 days and 12 days,respectively. B and C started the work and worked for 3 days and left. The number of days required by A alone to complete the remaining work, is: 

A 11 

B 15 ½

C 18 

D 13⅕

Answer: D 

 

Q. 59 If x y = 4 and xy = 45, then the value of x3 y3is:

A 822 

B 604 

C 151 

D 82 

Answer: B 

 

Q. 60 A person sells an article at 10% below its cost price. Had he sold it for ₹332 more, he would have made a profit of 20%. What is the original selling price (in ₹) of the article? 

A 996 

B 1328 

C 1028 

D 896 

Answer: A 

 

Q. 61 If the base radius of 2 cylinders are in the ratio 3 : 4 and their heights are in the ratio 4 : 9, then the ratio of their volumes is: 

A 2:1 

B 1:4 

C 1:2 

D 4:1 

Answer: B 

 

Q. 62 A train crosses a pole in 12 sec, and a bridge of length 170 m in 36 sec. Then the speed of the train is: 

A 10.8 km/h 

B 30.75 km/h 

C 25.5 km/h 

D 32.45 km/h 

Answer: C 

 

Q. 63 If ‘+’ means ‘-‘, ‘-‘ means ‘+’, ‘ ‘ means ‘ ‘, and ‘ ‘ means ‘ ‘, then the value of (42−12×3+8÷2+15)/ (8×2−4+9÷3) is:

A -¹⁵/₁₉

B ⁵/₃

C -⁵/₃

D ¹⁵/₁₉

Answer: A  

 

Q. 64 A shopkeeper marks the price of the article in such a way that after allowing a 28% discount, he wants a gain of 12%. If the marked price is ₹224. then the cost price of the article is: 

A ₹168 

B ₹144 

C ₹120 

D ₹196 

Answer: B 

 

Q. 65 ₹4,300 becomes ₹4,644 in 2 years at simple interest. Find the principle amount that will become ₹10,104 in 5 years at the same rate of interest. 

A ₹7,200 

B ₹8,420 

C ₹9,260 

D ₹5,710 

Answer: B 

 

Q. 66 Out of 6 numbers, the sum of the first 5 numbers is 7 times the 6th number. If their average is 136, then the  6th number is: 

A 116 

B 102 

C 96 

D 84 

Answer: B 

 

Q. 67 If A + B = 45, then the value of 2(1 + tan A)(1 + tan B)  is: 

A 4 

B 0 

C 2 

D 1 

Answer: A 

 

Q. 68 If the length of a rectangle is increased by 40%, and the breadth is decreased by 20%,then the area of the rectangle increases by x%. Then the value of x is: 

A 16 

B 20 

C 8 

D 12 

Answer: D 

 

Q. 69 The area of △ABC is 44cm2 . If D is the midpoint of BC and E is the mid point of AB,then the area (in cm2) of △BDE is: 

A 5.5 

B 44 

C 11 

D 22 

Answer: C 

 

Q. 70 The given table shows the number (in thousands) of cars of five different models A, B, C, D and E produced during Years 2012-2017. Study the table and answer the questions that follow. 

In the year 2015, which type of car constitutes exactly 20% of the total number of cars produced that year? 

A E 

B A 

C B 

D D 

Answer: A 

 

Q. 71 If A lies in the first quadrant and 6 tan A = 5, then the value of (8 sinA−4 cosA)/(cosA+2 sinA) is: 

A 16 

B 1 

C 4 

D -2 

Answer: B 

 

Q. 72 The ratio of the number of boys to the number of girls in a school of 640 students, is 5 : 3. If 30 more girls are admitted in the school, then how many more boys should be admitted so that the ratio of boys to that of the girls, becomes 14 : 9. 

A 30 

B 25 

C 15 

D 20 

Answer: D  

 

Q. 73 A, B and C are three points on a circle such that the angles subtended by the chord AB and AC at the centre O are 110and 130, respectively. Then the value of ∠BAC is: 

A 70

B 75

C 60

D 65

Answer: C 

 

Q. 74 The given table shows the number (in thousands) of cars of five different models A, B, C, D and E produced during Years 2012-2017. Study the table and answer the questions that follow. 

The percentage increase in the total cars in 2016 over 2012, is: 

A 50% 

B 45% 

C 33.33% 

D 62.33% 

Answer: A 

 

Q. 75 If x2a = y2b = z2c ≠ 0 and x2 = yz, then the value of  (ab+bc+ca)/ bc is: 

A 3 

B 3ac 

C 3bc 

D 3ab 

Answer: A 

 

English Comprehension 

Q. 76 Select the most appropriate option to substitute the underlined segment in the given sentence. If no substitution is required, select No improvement . 

The Director will agree with the proposal if we do not exceed the budget. 

A agree on a proposal 

B agree to the proposal 

C agreed by the proposal 

D No Improvement 

Answer: B 

 

Q. 77 Select the correct synonym of the given word. 

Obligatory 

A Mandatory 

B Reckless 

C Useless 

D Aggressive 

Answer: A 

 

Q. 78 Select the passive form of the given sentence. 

The manager keeps the work pending. 

A The work are being kept pending by the manager 

B The work was kept pending by the manager 

C The work has been kept pending by the manager 

D The work is kept pending by the manager. 

Answer: D 

 

Q. 79 In the sentence identify the segment which contains the grammatical error. 

One of the boys from our school have been selected for National Badminton Championship. 

A have been selected 

B from our school 

C One of the boys 

D for National Badminton Championship 

Answer: A 

 

Q. 80 Fill in the blank with the most appropriate word. 

We must ______ help to the homeless and physically disabled people. 

A exert 

B donate 

C contribute 

D render 

Answer: D 

 

Q. 81 Given below are four jumbled sentences. Select the option that gives their correct order. 

A. However, the rate of population increase is another important factor to consider.

B. This change can be expressed in two ways. 

C. Growth of population refers to the change in the number of inhabitants of a country.

D. First, in terms of absolute numbers and second, in terms of percentage change. 

A CADB 

B CBDA 

C BDCA 

D BADC 

Answer: B 

 

Instructions 

In the following passage some words have been deleted. Fill in the blanks with the 

help of the alternatives given. Select the most appropriate option for each blank. 

Communication plays a (1)______ role in the overall development of man. It can be learnt by our (2)______ efforts. Today, success in our professional life depends on our (3)______ to read, write and speak well which results in effective communication. Barriers (4)______ communication hinder the communication process. It is very important to (5)______ these barriers so that the transmission of the message can be smooth. 

Q. 82 Select the most appropriate option for blank No. 1 

A lifeless 

B total 

C vital 

D better 

Answer: C 

 

Q. 83 Select the most appropriate option for blank No. 2 

A conscious 

B contradictory 

C unclear 

D important 

Answer: A 

 

Q. 84 Select the most appropriate option for blank No. 3 

A ability 

B facility 

C agility 

D variety 

Answer: A 

 

Q. 85 Select the most appropriate option for blank No. 4 

A to 

B by 

C against 

D from 

Answer: A 

 

Q. 86 Select the most appropriate option for blank No. 5 

A strengthen 

B overcome 

C succeed 

D create 

Answer: B 

 

Instructions 

For the following questions answer them individually 

Q. 87 Select the word, which means the same as the groups of words given. 

A song sung at a burial 

A Dirge 

B Sonnet 

C Hymn 

D Ballad 

Answer: A 

 

Q. 88 Given below are four jumbled sentences. Select the option that gives their correct order 

A. However, they ignore the truth that progress and success are proportional to the labor they put in. 

B. The general human tendency is to find faults in the policies framed by the government. C. They blame the government for their slow progress, expecting miracles and magical transformation in their life. 

D. So people openly criticize and condemn the policy makers. 

A BDCA 

B DBAC 

C CDAB 

D ABCD 

Answer: A 

 

Q. 89 Select the correct synonym of the given word. 

Scintillating 

A Glittering 

B Stinging 

C Flattering 

D Boring 

Answer: A 

 

Q. 90 In the sentence identify the segment which contains the grammatical error. 

I can swim very fast when I was only five. 

A I can swim 

B only five 

C very fast 

D when i was 

Answer: A 

 

Q. 91 Select the correctly spelt word. 

A exhoust 

B exhibit 

C exteract 

D exhail 

Answer: B 

 

Q. 92 Select the appropriate meaning of the given idiom. 

A hard nut to crack 

A Not restrained 

B Easily encouraged 

C Easily disappointed 

D A difficult problem 

Answer: D 

 

Q. 93 Select the correct antonym of the given word. 

Quiescent 

A Dejected 

B Active 

C Peaceful 

D Indifferent 

Answer: B 

 

Q. 94 Select the indirect narration of the given sentence. 

He said to the hotel receptionist, “Can you tell me the tariff of rooms? 

A He enquired the hotel receptionist if he can tell him the tariff of rooms 

B He asked the hotel receptionist if he could tell him the tariff of rooms 

C He asked the hotel receptionist that if he can tell him the tariff of rooms 

D He asked the hotel receptionist to tell him the tariff of rooms 

Answer: B 

 

Q. 95 Select the correctly spelt word. 

A Bouquette 

B Sarcasm 

C Humilliation 

D Retalaite 

Answer: B 

 

Q. 96 Select the most appropriate option to substitute the underlined segment in the given sentence. If no substitution is required, select No improvement 

The captain as well the players were responsible for winning the trophy 

A The captain as well as the players was 

B No Improvement 

C As The captain with the players were 

D The captain also the players were 

Answer: A 

 

Q. 97 Select the appropriate meaning of the given idiom. 

To take French leave 

A Leave without any intimation 

B Welcome the host 

C Acknowledge the host 

D Leave with written permission 

Answer: A 

 

Q. 98 Fill in the blank with the most appropriate word. 

Handle this glass table with care because it is ______ 

A fragile 

B frugal 

C volatile 

D ductile 

Answer: A 

 

Q. 99 Select the word, which means the same as the given group of words. 

Something that cannot be heard 

A inaudible 

B infallible 

C irrevocable 

D audible 

Answer: A 

 

Q. 100 Select the correct antonym of the given word. 

Exodus 

A Exit 

B Refund 

C Arrival 

D Departure 

Answer: C 

SSC CHSL Tier-I 2 July 2019 Shift-III Previous Year Paper

SSC CHSL

(2 July 2019 Shift-III) 

Reasoning 

Instructions For the following questions answer them individually 

Q.1 Which number will replace the question mark (?) in the following series? 2, 4, 16, 52, 132, 282, ? 

A 534 

B 512 

C 572 

D 527 

Answer: A 

 

Q.2 If ‘FORTUNE’ is coded as ‘6521347’ and ‘PREY’ is coded as ‘8279’ then how will ‘NEPTUNE’ be coded as? 

A 4781343 

B 4781347 

C 4782347 

D 4781342 

Answer: B 

 

Q.3 Which number will replace the question mark (?) in the following series? 89, 90, 82, 109, 45, ? 

A 170 

B 125 

C -80 

D 109 

Answer: A 

 

Q.4 Select the Venn diagram that best illustrates the relationship between the following classes. Nurses, Doctors, Pharmacists 

Answer: B 

 

Q.5 Two statements are given, followed by three conclusions numbered I, II and III. Assuming the statements to be true, even if they seem to be at variance with commonly known facts, decide which of the conclusions logically follow(s) from the statements. Statements: All women are engineers. Some singers are engineers. 

Conclusions: I. All women are singers II. All singers are women III. Some engineers are women. 

A Only conclusion III follows. 

B Only conclusions II and III follow. 

C Only conclusion I follows. 

D Only conclusions I and II follow. 

Answer: A 

 

Q.6 Three of the following four numbers are alike in a certain way and one is different. Pick the number that is different from the rest. 

A 83115 

B 43217 

C 54215 

D 12347 

Answer: A 

 

Q.7 Select the set in which the numbers are related in the same way as are the numbers of the following set. (15, 102, 313) 

A (11, 74, 229) 

B (5, 32, 121) 

C (8, 53, 164) 

D (9, 60, 185) 

Answer: A 

 

Q.8 Select the correct mirror image of the given figure when the mirror is placed to the right of the figure. 

Answer: C 

 

Q.9 In a code language, ‘ CERTAIN ‘ is written as ‘XVIGZRM’. How will ‘CLIMATE’ be written as in that language? 

A XONRZGV 

B XORNZVG 

C XRONZGV 

D XORNZGV 

Answer: D 

 

Q.10 Select the figure that will come next in the following figure series. 

Answer: A

 

Q.11 Three different positions of the same dice are shown. Find the number on the face opposite to the one having 1. 

A 6 

B 2 

C 4 

D 3 

Answer: A 

 

Q.12 Select the combination of letters that when sequentially placed in the gaps of the given letter series will complete the series. ab_ba_db_bd_ac_ba_db_cdbabd_ 

A cdabdbba 

B dcdabbab 

C dcadbbab 

D dcabdbab 

Answer: D 

 

Q.13 From the given alternatives select the numbers that must be interchanged to make the following equation correct. 16 − 6 + 12 × 3 ÷ 4 = 18 

A 6 and 3 

B 6 and 4 

C 12 and 4 

D 16 and 3 

Answer: B

 

Q.14 Select the word-pair in which the two words are related in the same way as are the two words in the following word-pair. Myopic : Farsighted 

A Trouble : Grief 

B Postpone : Suspend 

C Famous : Obscure 

D Confusion : Chaotic 

Answer: C 

 

Q.15 Ranga got thrice as many sums wrong as he got right in a mathematics assignment. If he attempted 72 sums in all, how many did he solve correctly in his mathematics assignment? 

A 20 

B 14 

C 16 

D 18 

Answer: D 

 

Q.16 T is the sister of D. D is married to P. P is the son of M. T is the mother of J. Y is the father of U. Y has only one son and only one daughter. U is the daughter of T. Q is the son of D. If M is the wife of W, then how is Q related to W? 

A Son-in-law 

B Son 

C Grandson 

D Nephew 

Answer: C 

 

Q.17 How many triangles are present in the following figure ? 

A 29 

B 27 

C 23 

D 25 

Answer: B 

 

Q.18 Arrange the following words in a logical and meaningful order. 1. Lustrum 2. Century 3. Year 4. Millennium 5. Decade 

A 2, 1, 5, 3, 4 

B 5, 4, 3, 2, 1 

C 5, 1, 3, 2, 4 

D 3, 1, 5, 2, 4 

Answer: D 

 

Q.19 In a code language, CONSULTANCY is written as QOJKYOUYJWP. How will MASTERMINDS be written as in that language? 

A APOWJIOZJEI 

B APOWHIOZJEI 

C APOWIHOZJEI 

D APOWIIOZJEI 

Answer: D 

 

Q.20 Select the option in which the given figure is embedded. 

Answer: C 

 

Q.21 Three of the following four words are alike in a certain way and one is different. Pick the odd word out. 

A Italy 

B Austria 

C Germany 

D Singapore 

Answer: D 

 

Q.22 A paper is folded and cut as shown in the following figures. How will it appear when unfolded ? 

Answer: D

 

Q.23 ₹2100 is divided among P, Q and R in such a way that P’s share is half of the combined share of Q and R, and Q’s share is one-fourth of the combined share of P and R. By what amount is R’s share more than that of P? 

A ₹200 

B ₹250 

C ₹500 

D ₹280 

Answer: D 

 

Q.24 ‘Cone’ is related to ‘Ice-cream’ in the same way as ‘Bowl’ is related to ‘_________’. 

A Sip 

B Soap 

C Soup 

D Drink 

Answer: C 

 

Q.25 Select the option that is related to the third letter-cluster in the same way as the second letter-cluster is related to the first letter-cluster. BANK : WXLJ :: IDOL : ______ 

A DAKM 

B DAMK 

C DAMJ 

D ADMK 

Answer: B 

 

General knowledge 

Instructions For the following questions answer them individually 

Q.26 The ‘World Blood Donor Day’ was celebrated around the world on _____ 2019 with the theme ‘Safe blood for all’. 

A 25 thMay 

B 29 thApril 

C 16 thMarch 

D 14 th June 

Answer: D 

 

Q.27 The quote “The shots that hit me are the last nails to the coffin of British rule in India.” belongs to which of the following freedom fighters? 

A Lala Lajpat Rai 

B Chandrashekhar Azad 

C Ram Prasad Bismil 

D Bhagat Singh 

Answer: A 

 

Q.28 Who among the following was elected to the board of the International Air Transport Association (IATA) in June 2019? 

A Naresh Goyal 

B Ajay Singh 

C Rahul Bhatia 

D Bhaskar Bhat 

Answer: B 

 

Q.29 Former Flipkart executive Sandeep Patil was appointed as India Managing Director of which company in May 2019? 

A Snapdeal 

B TrueCaller 

C Paytm 

D Amazon 

Answer: B 

 

Q.30 In the field of Information & Communication Technology, what is the full form of EEPROM ? 

A Electrically Efficient Programmable Read-Only Memory 

B Enhanced Electrical Portable Read-Only Memory 

C Electrically Erasable Programmable Read-Only Memory 

D Electrically Efficient Portable Read-Only Memory 

Answer: C 

 

Q.31 Herts is the SI unit of: 

A Force 

B Frequency 

C Energy 

D Pressure 

Answer: B 

 

Q.32 What is the name of the Young Scientist Programme inaugurated by the Chairman of ISRO in May 2019? 

A Young India-2019 

B Yuva-2019 

C Yuvika-2019 

D Youth-2019 

Answer: C 

 

Q.33 Soorya Festival’ is associated to which of the following cities? 

A Chennai 

B Hyderabad 

C Thiruvananthapuram 

D Jaipur 

Answer: C 

 

Q.34 Who among the following clinched a gold medal in the women’s 25m pistol event at the 2019 ISSF World Cup? 

A Manu Bhaker 

B Heena Sidhu 

C Devanshi Rana 

D Rahi Sarnobat 

Answer: D 

 

Q.35 The winners of the 2019 ICC men’s World Cup will earn a cash award of ______ which is the highest prize money on offer in the history of the tournament. 

A $4 million 

B $2 million 

C $6 million 

D $5 million 

Answer: A 

 

Q.36 Financial Literacy Week 2019′ is an initiative by which bank to promote awareness on key topics every year through a focused campaign? 

A ICICI Bank 

B State Bank of India 

C Indian Bank 

D Reserve Bank of India 

Answer: D 

 

Q.37 The National Institute of Nutrition is located in: 

A Mumbai 

B Hyderabad 

C Indore 

D Shillong 

Answer: B 

 

Q.38 In which state is the Mundra Power Plant located? 

A Odisha 

B Madhya Pradesh 

C Gujarat 

D Chhattisgarh 

Answer: C 

 

Q.39 Which one of the following Union Territories has been granted with partial statehoodship? 

A Daman and Diu 

B Puducherry 

C Lakshadweep 

D Dadra and Nagar Haveli 

Answer: B 

 

Q.40 Which of the following countries hosted the Asian Cup Football tournament in 2019? 

A Singapore 

B UAE 

C South Korea 

D China 

Answer: B 

 

Q.41 The Himalayas belong to which of the following categories of mountains? 

A Fault-block Mountain 

B Fold Mountain 

C Dome Mountain 

D Plateau Mountain 

Answer: B 

 

Q.42 In the solar system, among the planets listed below, which planet is farthest from the Sun? 

A Neptune 

B Jupiter 

C Mars 

D Venus 

Answer: A 

 

Q.43 Which State Government increased the financial aid given to girls under the ‘Aapki Beti’ scheme in May 2019? 

A Sikkim 

B Odisha 

C Rajasthan 

D Assam 

Answer: C 

 

Q.44 In which state is Sambhar, the largest inland salt water lake of India located? 

A Maharashtra 

B Andhra Pradesh 

C Odisha 

D Rajasthan 

Answer: D 

 

Q.45 The investment and financial services major Goldman Sachs has sanctioned $250 million to construct one of its largest global workspaces in ______. 

A Chennai 

B Gurugram 

C Bengaluru 

D Pune 

Answer: C 

 

Q.46 In Lok Sabha 2019 Elections, Congress won _______ seats out of the 13 Lok Sabha seats of Punjab. 

A 8 

B 5 

C 6 

D 10 

Answer: A 

 

Q.47 Pivot Table’ is a feature of which of the following softwares? 

A Microsoft PowerPoint 

B Microsoft Access 

C Microsoft Word 

D Microsoft Excel 

Answer: D 

 

Q.48 Which eminent novelist and poet of the 20th century is the author of ‘Pinjar’? 

A Bhawani Prasad Mishra 

B Prabha Kiran Jain 

C Amrita Pritam 

D Harivansh Rai Bachchan 

Answer: C 

 

Q.49 In May 2019, which of the following companies announced that it would stop selling its smartphones in India? 

A OnePlus 

B Apple 

C Sony 

D Samsung 

Answer: C 

 

Q.50 In which year was Satya Shodhak Samaj founded by Jyotirao Phule? 

A 1865 

B 1861 

C 1870 

D 1873 

Answer: D 

 

Quantitative Aptitude 

Instructions For the following questions answer them individually 

Q.51 If x2 + 1 = 3x, then the value of (x4 +x−2 )/(x2+5x+1) is: 

A 3½

B 4½

C 2¼

D 2⅓

Answer: C 

 

Q.52 A circle is inscribed in a quadrilateral ABCD, touching sides AB, BC CD and DAatP, Q, R and §,respectively. If AS = 8 cm, BC = 11 cm, and CR=5 cm, then the length AB is equal to: 

A 14cm 

B 16cm 

C 12cm 

D 13cm 

Answer: A 

 

Q.53 A simplified value of [sinθ /(1+cosθ) + (1+cosθ)/sinθ )] 1/(tanθ+cotθ ) is: 

A 2sinθ

B cosθ

C sinθ

D 2cosθ

Answer: D 

 

Q.54 If 40√5x3 − 3√3 y3 = (2√5 x − √3 y) × (Ax2 + Bxy + Cy2 ), then what is the value of √(B2 + C2A)

A 9 

B 11 

C 7 

D 8 

Answer: C 

 

Q.55 The given Bar Graph presents the Demand and Production of motorcycles of five companies (in lakhs). 

The total Production of motorcycles of companies B and D taken together is what percent of the Demand of motorcycles of all the companies taken together? 

A 48% 

B 40% 

C 46% 

D 38% 

Answer: C 

 

Q.56 The value of [(cos9 +sin81 )(sec 9 +cosec 81 )] /sin56 sec 34 +cos25 cosec 65 is: 

A 1 

B 4 

C 2 

D ½ 

Answer: C 

 

Q.57 The radius of the base of a cylinder is 7 cm and its curved surface area is 440 cm² . Its volume (in cm³ ) 

will be: (Take π = ²²/₇) 

A 1760 

B 1430 

C 1540 

D 1650 

Answer: C 

 

Q.58 Anu spends 90% of her income. If her expenditure increases by 25% and savings increase by 30%, then by what percent does hersalary increase? 

A 22.5% 

B 20% 

C 24% 

D 25.5% 

Answer: D 

 

Instructions: The given Bar Graph presents the Demand and Production of motorcycles of five companies (in lakhs). 

Q.59 The company in which the Production of motorcycles is approximately 23% more than the Demand is: 

A D 

B B 

C E 

D C 

Answer: D 

 

Q.60 What is the ratio of the total Demand of motorcycles of companies A and D taken together to the Production of motorcycles of company C? 

A 5 : 3 

B 8 : 5 

C 13 : 9 

D 9 : 7 

Answer: A

 

Instructions For the following questions answer them individually 

Q.61 If 3 − 2sin2 θ − 3cosθ = 0,0 < θ < 90 , then what is the value of (2cosecθ + tanθ) ? 

A 7√3 

B 5√3 

C 5√3/3

D 7√3/3 

Answer: D 

 

Q.62 The total number of students in class A and B is 96. The number of students in A is 40% more than that in B. The average weight (in kg) of the students in B is 50% more than that of the students in A. If the average weight of all the students in A and taken together is 58 kg, then what is the average weight of the students in B? 

A 48 kg 

B 60 kg 

C 66 kg 

D 72 kg 

Answer: D 

 

Q.63 Abhi sold two articles for ₹5,220 each. On one, he gained 16% and on the other, he lost 10%. His profit or loss on the whole was: 

A Loss, ₹130 

B Profit, ₹180 

C Profit, ₹140 

D Loss, ₹125 

Answer: C

 

Q.64 In △ABC,  ∠A = 90°. If BL and CM are the medians, then: 

A 4(BL2 + CM2) = 5BC2

B 3(BL2+ CM2 ) = 4BC2 

C 4(BL2 + CM 2) = 3BC2

D 5(BL2 + CM2 ) = 4BC2 

Answer: A 

 

Q.65 If a + b + c = 5 , and a2 + b2 + c2 = 33, then what is the value of a3 +b3 + c3 − 3abc

A 195 

B 185 

C 180 

D 192 

Answer: B 

 

Q.66 What is the compound interest on a sum of ₹4,096 at 15% p.a. for 2½years, if the interest is compounded 10-monthly? 

A ₹1,736 

B ₹1,763 

C ₹1,636 

D ₹1,726 

Answer: A 

 

Q.67 In a circle, chords AB and CD intersect each other at E. If CD = 18 cm, DE = 6 cm and AE = 18 cm, then BE = ? 

A 3 cm 

B 4 cm 

C 8 cm 

D 6 cm 

Answer: B 

 

Q.68 If 3cos2 A + 6sin2 A = 3,0A ≤ 90 , then the value of A is: 

A 45 

B 30 

C 90 

D 0

Answer: D 

 

Q.69 Let △ABC ∼ △QPR and ar(△ABC)/ar(△PQR) = 9/16 . If AB = 12 cm, BC = 6 cm and AC = 9 cm, then QP is equal to: 

A 16 cm 

B 8 cm 

C 9 cm 

D 12 cm 

Answer: A 

Q.70 A train x running at 84 km/h crosses another train y running at 52 km/h in opposite direction in 12 seconds. If the length of y is two-third that of x, then what is the length of x? 

A 408 m 

B 250 m 

C 272 m 

D 242 m 

Answer: C 

 

Q.71 The given Bar Graph presents the Demand and Production of motorcycles of five companies(in lakhs). 

The average Production of motorcycles of companies B, C and E taken together is what percent less than the Demand of motorcycles of D ? 

A 6% 

B 9.3% 

C 8% 

D 8.7% 

Answer: C 

 

Q.72 If a3 + b3 = 110 and a + b = 5 , then (a + b)2 − 3ab is equal to: 

A 42 

B 32 

C 22 

D 52 

Answer: C 

 

Q.73 In an examination, the success to failure ratio was 5 : 2. Had the number of failures been 14 more, then the success to failure ratio would have been 9 : 5. The total number of candidates who appeared for the examination was: 

A 126 

B 210 

C 196 

D 203 

Answer: C 

 

Q.74 A and B, working together, can complete a work in 16 days, C and A together can complete it in 32 days, B and C together can complete it in 24 days. They worked togetherfor 12 days. In how many days will C alone complete the remaining work ? 

A 36 

B 32 

C 45 

D 40 

Answer: A 

 

Q.75 In △ ABC, AD BC at D and AE is the bisector of ∠ B = 72 and ∠ C = 26 , then what is the measure of ∠ DAE ? 

A 25 

B 37 

C 23 

D 49 

Answer: C 

 

English 

Instructions For the following questions answer them individually 

Q.76 Select the most appropriate option to substitute the underlined segment in the given sentence. If there is no need to substitute it, select No improvement. The children are too young to sit up a long lecture. 

A sit across 

B No improvement 

C sit away 

D sit through 

Answer: D 

 

Q.77 Select the wrongly spelt word. 

A emporium 

B emolument 

C emmision 

D emphasis 

Answer: C 

 

Q.78 Select the most appropriate meaning of the given idiom. Spill the beans 

A frighten someone 

B create difficulty 

C reveal a secret 

D make an excuse 

Answer: C 

 

Q.79 Select the word which means the same as the group of words given. That which cannot be taken by force 

A inaccessible 

B impregnable 

C inapprehensible 

D immutable 

Answer: B 

 

Q.80 Select the word which means the same as the group of words given. A large impressive house 

A cabin 

B fort 

C shack 

D mansion 

Answer: D 

 

Q.81 Select the most appropriate antonym of the given word. CORROBORATE 

A contradict 

B confirm 

C approve 

D verify 

Answer: A 

 

Q.82 In the sentence identify the segment which contains the grammatical error. The Cannes Film Festival attract some of the world’s most famous people. 

A the world’s 

B most famous people 

C attract some of 

D The Cannes Film Festival 

Answer: C 

 

Q.83 Select the most appropriate antonym of the given word. MOROSE 

A cheerful 

B unsatisfied 

C mournful 

D sullen 

Answer: A 

 

Q.84 Select the correct passive form of the given sentence. Bill Gates has given away twenty seven percent of his wealth in charity. 

A Bill Gates has been given away by twenty seven percent of his wealth in charity. 

B Twenty seven percent of his wealth has given away Bill Gates in charity. 

C Twenty seven percent of his wealth has been given away by Bill Gates in charity. 

D Twenty seven percent of his wealth had been given away by Bill Gates in charity. 

Answer: C 

 

Q.85 Select the correct indirect form of the given sentence. I asked the shopkeeper, “What is the price of this bike?” 

A I asked the shopkeeper that what the price of this bike was. 

B I asked the shopkeeper what is the price of this bike. 

C I asked the shopkeeper what the price of that bike was. 

D I asked the shopkeeper what was the price of that bike. 

Answer: C 

 

Q.86 Given below are four jumbled sentences. Out of the given options pick the one that gives their correct order. 

A. In a small town in England, there lived a poor little girl. 

B. She was very excited but her right shoe was worn out. 

C. So, she went to a cobbler to get it mended. 

D. Her friend invited her to come to her birthda y party. 

A ACBD 

B CBAD 

C ADBC 

D DCAB 

Answer: C 

 

Q.87 In the sentence identify the segment which contains the grammatical error. Different-coloured natural foods not only put a smile on children’s faces and also benefit their overall growth. 

A put a smile on children’s faces 

B Different-coloured natural foods 

C benefit their overall growth 

D and also benefit 

Answer: D 

 

Q.88 Select the most appropriate word to fill in the blank. Financial institutions that had their premises in the World Trade Centre were crippled by the loss of ______ and hardware. 

A personnel 

B group 

C individual 

D person 

Answer: A 

 

Q.89 Given below are four jumbled sentences. Out of the given options pick the one that gives their correct order. 

A. Early on Christmas morning Pip returns to the graveyard with the file, a pie and brandy. B. On Christmas Eve, Pip, an orphan who is about seven years old, encounters an escaped convict in the village graveyard. 

C. The convict scares Pip into st ealing food and a file. 

D. Pip, who lives with his elder sister and her husband Joe Gargery, a blacksmith, steals a file and some food. 

A CBAD 

B BDCA 

C DCAB 

D BCDA 

Answer: D 

 

Q.90 Select the most appropriate meaning of the given idiom. As clear as mud 

A extremely dirty 

B honest and legal 

C completely suitable for someone 

D impossible to understand 

Answer: D 

 

Q.91 Select the most appropriate synonym of the given word. COMBUSTIBLE 

A inflammable 

B fragile 

C nonexplosive 

D unbreakable 

Answer: A 

 

Q.92 Select the most appropriate word to fill in the blank. The thick smoke ______ out of the landfill site is making the city’s polluted air even more toxic. 

A billowing 

B burning 

C rocking 

D tossing 

Answer: A 

 

Q.93 Select the most appropriate synonym of the given word. EMBED 

A pull 

B rise 

C bury 

D dig up 

Answer: C 

 

Q.94 Select the most appropriate option to substitute the underlined segment in the given sentence. If there is no need to substitute it, select No improvement. I try to avoid travel in the rush hour. 

A No improvement 

B travelling in 

C to travel at 

D travelling on 

Answer: B 

 

Instructions: In the following passage some words have been deleted. Fill in the blanks with the help of the alternatives given. Select the most appropriate option for each blank. 

Comprehension: Bill Gates is the second-richest (1) ______ in the world. He is (2) ______ an estimated $ 103 billion. His wealth (3) ______ only by Jeff Bezos, (4) ______ has a current net worth of $116 billion. Gates is (5) ______ rich that an average American spending $1 is similar to Gates spending $ 1.06 million. 

Q.95 Select the most appropriate option for blank No. 1. 

A personnel 

B character 

C human 

D person 

Answer: D 

 

Q.96 Select the most appropriate option for blank No. 2. 

A valuable 

B classed 

C worth 

D cost 

Answer: C 

 

Q.97 Select the most appropriate option for blank No. 3. 

A is surpassed 

B surpassed 

C is surpassing 

D surpasses 

Answer: A 

 

Q.98 Select the most appropriate option for blank No. 4. 

A whose 

B whom 

C which 

D who 

Answer: D 

 

Q.99 Select the most appropriate option for blank No. 5. 

A as 

B so 

C too 

D such 

Answer: B 

 

Instructions For the following questions answer them individually 

Q.100 Select the wrongly spelt word. 

A brood 

B booty 

C boast 

D blosom 

Answer: D 

SSC CHSL Tier-I 2 July 2019 Shift-II Previous Year Paper

SSC CHSL

(2 July 2019 Shift-II) 

Reasoning 

Instructions For the following questions answer them individually 

Q.1 Arrange the following words in a logical and meaningful order. 

1. Results  2. Campaign 3. Sworn  4. Nominations 5. Polling 

A 5, 2, 4, 1, 3 

B 4, 5, 2, 1,3 

C 4, 2, 5, 1, 3 

D 5, 4, 2, 1, 3 

Answer: C 

 

Q.2 If PEAR is coded as 7519 and TOIL is coded as 2693, then how will DOCTOR be coded ? 

A 463239 

B 463729 

C 463296 

D 463269 

Answer: D 

 

Q.3 Select the option that is related to the third letter-cluster in the same way as the second letter-cluster is related to the first letter-cluster. LENOVO : EKISSQ :: FINGER : ________ 

A YOIKCT 

B YOIKBT 

C YNIKBT 

D YOIJBT 

Answer: B 

 

Q.4 In a code language, TABLEPADS is written as BADCYGAPJ. How will BACKSPACE be written as in that language? 

A DAEBJGAEY 

B DEIBJGEIY 

C DAIBJGAIY 

D DAJBJGAJY 

Answer: C 

 

Q.5 The sum of the squares of three numbers is 336 and the ratio of the first and the second as also of the second and the third is 1 : 2. Then find the difference of third and first number? 

A 10 

B 12 

C 14 

D 16 

Answer: B 

 

Q.6 If MARBLES is coded as 181132192512 and INVITED is coded as 221969942520, then how will COMPARE be coded as? 

A 1322531253221 

B 1322531652241 

C 1322531653241 

D 1312531653241 

Answer: C 

 

Q.7 Select the set in which the numbers are related in the same way as are the numbers of the following set. (8, 101, 35) 

A (6, 76, 27) 

B (11, 136, 47) 

C (9, 121, 39) 

D (7, 89, 31) 

Answer: D 

 

Q.8 Select the Venn diagram that best illustrates the relationship between the following classes. Mothers, Women, Pilots 

Answer: A 

 

Q.9 Which number will replace the question mark (?) in the following series? 4, 10, 27, 140, ? 

A 379 

B 397 

C 973 

D 937 

Answer: C 

 

Q.10 Select the option that is related to the third number in the same way as the second number is related to the first number. 81 : 63 :: 74 : _____ 

A 33 

B 13 

C 43 

D 23 

Answer: A 

 

Q.11 Select the correct mirror image of the given figure when the mirror is placed to the right of the figure. 

Answer: A 

 

Q.12 In a certain code, SLUMBER is coded as PORPYHO. How would PURPOSE be written in that same code? 

A MXOSLVB 

B OTQONRD 

C SRUMRPG 

D QVSQPTF 

Answer: A 

 

Q.13 Select the term that will come next in the following series. 3, 27, 243, 2187, ___ 

A 13561 

B 21377 

C 27139 

D 19683 

Answer: D

 

Q.14 Select the combination of letters that when sequentially placed in the gaps of the given letter series will complete the series. ab_bcab_b__ab_bcab_bc 

A dcdcd 

B cdcdc 

C ccdcc 

D ddcdd 

Answer: D 

 

Q.15 How many triangles are present in the following figure ? 

A 18 

B 20 

C 14 

D 16 

Answer: A 

 

Q.16 Gopal’s age is the cube of a number. It was the square of a number two years ago. What is his age? 

A 9 

B 62 

C 64 

D 27 

Answer: C 

 

Q.17 Three different positions of a dice are shown below. Which number appears on the face opposite the number 6 ? 

A 3 

B 1 

C 4 

D 5 

Answer: C 

 

Q.18 Select the word-pair in which the two words are related in the same way as are the two words in the following word-pair. Candle : Illuminate 

A Refrigerator : Cool 

B Oven : Grind 

C Fan : Suffocation 

D Bulb : Dark 

Answer: A 

 

Q.19 Three of the following four numbers are alike in a certain way and one is different. Pick the number that is different from the rest. 

A 6394 

B 2936 

C 4638 

D 3864 

Answer: A 

 

Q.20 Select the option in which the given figure is embedded. 

Answer: A 

 

Q.21 Select the figure that will come next in the following figure series. 

Answer: D 

 

Q.22 ‘Iraq’ is related to ‘Dinar’ in the same way as ‘Myanmar’ is related to ‘_______’. 

A Lira 

B Rand 

C Yen 

D Kyat 

Answer: D

 

Q.23 A paper is folded and cut as shown below. How will it appear when unfolded ? 

Answer: D 

 

Q.24 Select the set in which the numbers are related in the same way as are the numbers of the following set. (3, 13, 196) 

A (5, 15, 350) 

B (6, 16, 569) 

C (4, 14, 441) 

D (2, 12, 287) 

Answer: A 

 

Q.25 Three of the following four letter-clusters are alike in a certain way and one is different. Pick the odd one out. 

A AWDZ 

B UQJF 

C OKPL 

D IDWS 

Answer: D 

 

General knowledge 

Instructions: For the following questions answer them individually 

Q.26 Which of the following gases has an odour that is similar to the odour of rotten eggs? 

A Ammonia 

B Chlorine 

C Hydrogen Sulfide 

D Dimethylcyclohexane 

Answer: C 

 

Q.27 Name the element that must be present in a compound to classify it as an organic compound. 

A Zinc 

B Iron 

C Carbon 

D Ammonia 

Answer: C 

 

Q.28 Which of the following rivers is popularly known as ‘Dakshin Ganga’? 

A Periyar 

B Godavari 

C Krishna 

D Tungabhadra 

Answer: B 

 

Q.29 In December 2016, the National Payments Corporation of India developed a mobile payment app named as__________ 

A NAKUL 

B JANAK 

C GANDIV 

D BHIM 

Answer: D 

 

Q.30 In 1872, Lord Mayo, the Viceroy of India was assassinated in ___________ 

A Kolkata 

B Diu 

C Delhi 

D Port Blair 

Answer: D 

 

Q.31 In the world of computers, what is ‘Ubuntu’? 

A Latest financial software package by Infosys 

B A firewall developed by Microsoft 

C An ERP system from SAP 

D Open source Linux operating system 

Answer: D 

 

Q.32 The famous Dwarkadhish Temple of Gujarat is located on the banks of which of the following rivers? 

A Luni 

B Tapti 

C Narmada 

D Gomti 

Answer: D 

 

Q.33 The 124th Amendment Bill, 2019 deals with: 

A Economic Reservation 

B Triple Talaq 

C GST 

D Privatisation of Air India 

Answer: A 

 

Q.34 The mother of Mahatma Gandhi was: 

A Heerabai 

B Putlibai 

C Kamlabai 

D Kasturbabai 

Answer: B 

 

Q.35 Ram Vilas Paswan is the president of which party? 

A Republican Party of India 

B Samajwadi Party 

C Lok Janshakti Party 

D Bahujan Samaj Party 

Answer: C 

 

Q.36 Which of the following cities houses the headquarters of the Union Bank of India? 

A Kolkata 

B Lucknow 

C Mangalore 

D Mumbai 

Answer: D 

 

Q.37 After the swearing in of the new Union government in May, who of the followings was allocated the portfolio of Youth Affairs and Sports? 

A Smriti Irani 

B K Muralidharan 

C Kiren Rijiju 

D Pratap Chandra Sarangi 

Answer: C 

 

Q.38 In the context of international banking transactions what is the full form of SWIFT? 

A Society for Worldwide International Financial Transactions 

B Society for Worldwide International Financial Telecommunication 

C Society for Worldwide Interbank Financial Telecommunication 

D Society for Wireless International Financial Transactions 

Answer: C 

 

Q.39 Greta Thunberg, the teenage sensation and global climate change crusader is from which of the following European countries ? 

A Sweden 

B Finland 

C Belgium 

D Germany 

Answer: A 

 

Q.40 In computer terminology, what is the full form of RAM? 

A Regular Access Memory 

B Repeated Access Memory 

C Rapid Access Memory 

D Random Access Memory 

Answer: D 

 

Q.41 Which colour saree embellished with bright golden borders is worn by the dancers of Mohiniyattam? 

A Deep red /red 

B White/off-white 

C Sea blue /blue 

D Parrot green /green 

Answer: B 

 

Q.42 Which of the following activities by Benjamin Franklin led him to gain a better understanding of lightning and eventually to the invention of the lightning rod? 

A Practising on a piano 

B Swimming in a river 

C Flying a kite 

D Riding a horse 

Answer: C 

 

Q.43 The 72nd Annual Cannes Film Festival 2019 was held in ______. 

A Italy 

B France 

C United Kingdom 

D Russia 

Answer: B 

 

Q.44 Who is the Vice Captain of the Indian team that is playing the World Cup 2019 in England ? 

A Rohit Sharma 

B Hardik Pandya 

C MS Dhoni 

D Shikhar Dhawan 

Answer: A 

 

Q.45 Which of the following types of bank accounts does NOT earn any interest for the account holder? 

A Pension Account 

B Recurring Deposit Account 

C Current Account 

D Savings Account 

Answer: C 

 

Q.46 Bangladesh defeated __________in their very first match of ODI(One Day International) World Cup held in England. 

A West Indies 

B New Zealand 

C Pakistan 

D South Africa 

Answer: D 

 

Q.47 The city of Belagavi is located in the state of: 

A Kerala 

B Karnataka 

C Gujarat 

D Tamil Nadu 

Answer: B 

 

Q.48 Famous writer and Congress leader Shashi Tharoor won the 2019 Lok Sabha Elections from which of the following constituencies of Kerala? 

A Thiruvananthapuram 

B Alleppey 

C Thrissur 

D Kozhikode 

Answer: A 

 

Q.49 Who of the following was elected as the Leader of Congress Parliamentary Party after the 2019 Lok Sabha elections? 

A Mallikarjun Kharge 

B Sonia Gandhi 

C Rahul Gandhi 

D Shashi Tharoor 

Answer: B 

 

Q.50 Which of the following banks was authorised to issue Electoral Bonds? 

A HDFC Bank 

B Corporation Bank 

C ICICI Bank 

D State Bank of India 

Answer: D 

 

Quantitative Aptitude 

Instructions For the following questions answer them individually 

Q.51 If 3sinθ = 2cos2 θ,0 < θ < 90 , then the value of (tanθ + cosθ + sinθ) is: 

A (3+5√3)/6 

B 5√3/3

C (3+5√3)/3 

D 5√3/6 

Answer: A 

 

Q.52 PAT is a tangent to a circle at point A on it, and AB is a chord such that ∠BAT = 72 . If C is a point on the circle such that ∠CBA = 58 , then what is the measure of ∠CAB

A 60 

B 48 

C 62 

D 50 

Answer: D 

 

Q.53 If 1/(secθ−tanθ) − 1/cosθ = secθ × k,0 < θ < 90 , then k is equal to: 

A cosecθ 

B tanθ 

C cotθ 

D sinθ 

Answer: D 

 

Q.54 The ratio of the present ages of A and is 6 : 5. Four years ago,this ratio was 5 : 4. What will be the ratio of the ages of A and after 12 years from now ? 

A 8:7 

B 3:2 

C 7:6 

D 9:8 

Answer: D 

 

Q.55 Amit travelled from A to B at an average speed of 80 km/h. He travelled the first 75% of the distance in two-third of the time and the rest at a constant speed of x km/h. The value of is: 

A 56 

B 54 

C 60 

D 64 

Answer: C 

 

Q.56 In △ABC, the bisectors of ∠B and ∠C intersect each other at a point D. If ∠BDC = 104 , then the measure of ∠A is: 

A 30 

B 28 

C 32 

D 26 

Answer: B 

 

Q.57 The value of 3 × 2 ÷ 3 of 12 − 3 ÷ 2 × (2 − 3) × 2 + 3 ÷ 2 of 3 is: 

A 2⅓

B −3⅔

C −2⅓

D 3⅔ 

Answer: D 

 

Q.58 A sum of ₹7,500 amounts to ₹8,748 after 2 years at a certain compound interest rate per annum. What will be the simple interest on the same sum for 4⅗ years at double the earlier interest rate ? 

A ₹2,760 

B ₹5,520 

C ₹4,140 

D ₹8,180 

Answer: B 

 

Q.59 If the nine-digit number 8175x45y2 is divisible by 72, then the value of √(4x + y) , for the largest value of y, is: 

A 6 

B 8 

C 4 

D 5 

Answer: D 

 

Q.60 The curved surface area and the volume of a cylindrical pole are 132 m2 and 528 m3 , respectively. What is the height (in m) of the pole? (Take π = 22/7) 

A 3⅝

B 2½

C 2⅝

D 3½ 

Answer: C

 

Q.61 The given Bar graph presents the number of students from Science and Arts streams from a school in different years.

In which year the number of Arts students is 30% more than that of Science ? 

A 2012 

B 2013 

C 2011 

D 2014 

Answer: A 

 

Q.62 ABCD is a cyclic quadrilateral such that AB is a diameter of the circle circumscribing it and ∠ADC = 158. Then ∠BAC is equal to: 

A 68 

B 40 

C 38 

D 50 

Answer: A 

 

Q.63 If a + b + c = 4 and ab + bc + ca = 1 , then the value of a3 + b3 + c3 − 3abc is: 

A 60 

B 52 

C 50 

D 47 

Answer: B

 

Q.64 If cosec31 = x, then sin2 59 + 1/cosec2 31 + tan2 591/sin2 59 cosec2 59 is equal to: 

A x – 1 

B x2 + 1 

C x2 − 1 

D x + 1 

Answer: C 

 

Q.65 The simplified value of [ (sin225 + sin265)/(cos224 +cos266) + sin2 71 + cos71 sin19] is: 

A 1 

B 0 

C 3 

D 2 

Answer: D 

 

Q.66 If 24√3 x3 + 2√2 y3 = (2√3 x + √2 y)(Ax2 + Bxy + Cy2 ) then (2A + √6 B C) is equal to: 

A 6 

B 8 

C 14 

D 10 

Answer: D 

 

Q.67 In a class of 80 students, 60% participate in games and the rest do not. The average weight of the former group is 5% more than that of the latter. If the average weight to fall the students is 5121 kg, then what is the average weight (in kg) of the former group ? 

A 52.5 

B 54.5 

C 57.6 

D 60 

Answer: A 

 

Q.68 The given Bar graph presents the number of students from Science and Arts streams from a school in different years.

What is the ratio of the total number of Science students in 2011 and 2015 to that of Arts in 2012 and 2015 ? 

A 11 : 12 

B 8 : 9 

C 9 : 10 

D 12 : 13 

Answer: D 

 

Q.69 When an article is sold for ₹291, there is a loss of 3%. What will be the selling price of the article, if it is sold at a gain of 8%? 

A ₹324 

B ₹308 

C ₹316 

D ₹332 

Answer: A 

 

Instructions: The given Bar graph presents the number of students from Science and Arts streams from a school in different years.

Q.70 The average number of Science students in 2011, 2013 and 2015 is what percentage more than the number of Arts students in 2011 ? 

A 9⅟₁₁% 

B 9³/₁₁% 

C 8⅟₉% 

D 11⅟₉% 

Answer: A 

 

Q.71 The total number of Arts students in 2011, 2013 and 2015 is what percentage less than that of Science In the given five years (correct to one decimal place)? 

A 33.6% 

B 34.2% 

C 31.4% 

D 32.8% 

Answer: A 

 

Instructions For the following questions answer them individually 

Q.72 A, B and C can finish a task in 42 days, 84 days and 28 days, respectively. A started the work. B joined him after 3 days. If C joined them after 5 days from the beginning, then for how many days did A work till the completion of the task? 

A 18 

B 20 

C 15 

D 17 

Answer: D

 

Q.73 In a constituency, 40% of the voters are senior citizens. 40% of the senior citizen voters are illiterates and 25% of the non-senior citizen voters are literates. By what percentage is the number of literate senior citizens voters less than that of illiterate non-senior citizen voters ? 

A 46⅔

B 48⅓ 

C 40 

D 50 

Answer: A 

 

Q.74 In △ABC, D and E are the points on sides AC and AB, respectively, such that ∠ADE = ∠B. If AD =7.6 cm, AE = 7.2 cm, BE = 4.2 cm and BC = 8.4 cm,then DE is equal to: 

A 5.8 cm 

B 5.6 cm 

C 7.4 cm 

D 6.3 cm 

Answer: B 

 

Q.75 The vertices A, B, C and D of a quadrilateral ABCD lie on a circle. ∠A is thrice ∠C and ∠D is twice ∠B. What is the difference between the measures of ∠B and ∠C ? 

A 15 

B 28 

C 18 

D 20 

Answer: A 

 

English 

Instructions For the following questions answer them individually 

Q.76 Select the most appropriate antonym of the given word. JEOPARDIZE 

A threaten 

B hazard 

C imperil 

D protect 

Answer: D 

 

Q.77 In the sentence identify the segment which contains the grammatical error. On September 11, 2001, two hijacked aircrafts crashed to the World Trade Centre in New York. 

A World Trade Centre in New York 

B crashed to the 

C two hijacked aircrafts 

D On September 11, 2001 

Answer: B 

 

Q.78 Given below are four jumbled sentences. Out of the given options pick the one that gives their correct order. 

A. In the opening scene, the young boy Pip meets an escaped convict in a graveyard. 

B. ‘Great Expectations’ is a novel by Charles Dickens. 

C. The novel is set in Kent and London in the early 19th century. 

D. It depicts the personal growth and development of an orphan named Pip. 

A DCAB 

B BCDA 

C ABCD 

D CBAD 

Answer: B 

 

Q.79 Select the word which means the same as the group of words given. impossible or extremely difficult to understand 

A incompatible 

B incomprehensible 

C inaudible 

D inevitable 

Answer: B 

 

Q.80 Select the most appropriate meaning of the given idiom. tit for tat 

A to take advantage of someone 

B to make someone angry 

C to do harm as done to you 

D to reward people for the good done 

Answer: C 

 

Q.81 Select the most appropriate option to substitute the underlined segment in the given sentence. If there is no need to substitute it, select No improvement. The boys were delighted on the idea of going to Goa. 

A on an idea of 

B No improvement 

C at an idea of 

D at the idea of 

Answer: D 

 

Q.82 Select the correct passive form of the given sentence. The Japanese Emperor Akihito abdicated the Chrysanthemum Throne in April, 2019. 

A The Chrysanthemum Throne is abdicated by the Japanese Emperor Akihito in April,2019. 

B The Chrysanthemum Throne was abdicated by the Japanese Emperor Akihito in April,2019. 

C The Japanese Emperor Akihito was abdicated by the Chrysanthemum Throne in April, 2019. 

D The Chrysanthemum Throne has abdicated by the Japanese Emperor Akihito in April,2019. 

Answer: B 

 

Q.83 In the sentence identify the segment which contains the grammatical error. Billionaire Philip is the owner of the luxury yacht Lionheart, which worth an estimated $150 million. 

A the owner of 

B an estimated $150 million 

C which worth 

D Billionaire Philip is 

Answer: C 

 

Q.84 Select the word which means the same as the group of words given. a type of dome-shaped shelter built from blocks of solid snow 

A chalet 

B wigwam 

C igloo 

D hut 

Answer: C 

 

Q.85 Select the most appropriate option to substitute the underlined segment in the given sentence. If there is no need to substitute it, select No improvement. He tried to prevent his friend to smoke. 

A No improvement 

B smoking 

C from smoking 

D to smoking 

Answer: C 

 

Q.86 Select the correct indirect form of the given sentence. The shopkeeper asked me, “Where is your car?” 

A The shopkeeper asked me where is your car. 

B The shopkeeper asked me where was my car. 

C The shopkeeper asked me where my car was. 

D The shopkeeper asked me where my car is. 

Answer: C 

 

Q.87 Given below are four jumbled sentences. Out of the given options pick the one that gives their correct order. 

A. One fine morning, a hunter was getting ready to go hunting. 

B. He pulled the blanket over the sleeping child. 

C. As he did so, he thought the blanket was not warm enough. 

D. Before departing, he went to see his little baby who was sleeping in a crib. 

A ADBC 

B CBAD 

C ACBD 

D DCAB 

Answer: A 

 

Q.88 Select the most appropriate word to fill in the blank. There were not enough tinsmiths in Vienna to ______ the lanterns for street lighting. 

A compose 

B execute 

C accomplish 

D manufacture 

Answer: D 

 

Q.89 Select the wrongly spelt word. 

A curse 

B curry 

C curious 

D cursore 

Answer: D 

 

Instructions: In the following passage some words have been deleted. Fill in the blanks with the help of the alternatives given. Select the most appropriate option for each blank. 

Comprehension: Trade on the New York Stock Exchange was (1) ______ to a standstill on an autumn day (2) ______ 1994. The culprit for the outage (3) ______ a squirrel that had chewed through a power line near the computer centre of the (4) ______ in Trumbull, Connecticut. The shutdown (5) ______ lasted for half-an- hour, resulted in the NYSE operating at 85 percent capacity. 

Q.90 Select the most appropriate option for blank No. 1. 

A taken 

B worked 

C happened 

D brought 

Answer: D 

 

Q.91 Select the most appropriate option for blank No. 2. 

A on 

B by 

C at 

D in 

Answer: D 

 

Q.92 Select the most appropriate option for blank No. 3. 

A was 

B has 

C were 

D is 

Answer: A 

 

Q.93 Select the most appropriate option for blank No. 4. 

A exchange 

B town 

C store 

D bank 

Answer: A 

 

Q.94 Select the most appropriate option for blank No. 5. 

A whom 

B which 

C who 

D what 

Answer: B 

 

Instructions: For the following questions answer them individually 

Q.95 Select the most appropriate word to fill in the blank. Municipal corporations have started to take ______ of the problems of waste management. 

A cognizance 

B knowledge 

C perception 

D attention 

Answer: A 

 

Q.96 Select the most appropriate meaning of the given idiom. bite your tongue 

A stop yourself from saying something 

B be impossible to be understood 

C talk for a long time 

D get bruises in the mouth 

Answer: A 

 

Q.97 Select the most appropriate antonym of the given word. INORDINATE 

A undue 

B reasonable 

C exorbitant 

D extreme 

Answer: B 

 

Instructions: Select the most appropriate synonym of the given word. 

Q.98 ADHERE 

A comply 

B release 

C detach 

D ignore 

Answer: A 

 

Q.99 CURRENT 

A present 

B old 

C uncommon 

D antiquated 

Answer: A 

 

Instructions: For the following questions answer them individually 

Q.100 Select the wrongly spelt word. 

A satiate 

B settle 

C session 

D satallite 

Answer: D 

SSC CHSL Tier-I 2 July 2019 Shift-I Previous Year Paper

SSC CHSL

(2 July 2019 Shift-I) 

Reasoning 

Instructions For the following questions answer them individually 

Q.1 Two different positions of the same dice are shown. Find the number opposite to the face having 4. 

A 2 

B 3 

C 1 

D 6 

Answer: A 

 

Q.2  Select the option in which the given figure is embedded. 

Answer: D 

 

Q.3 Select the combination of letters that when sequentially placed in the gaps of the given letter series will complete the series. ab_ba_babd_acb_bdba_b 

A dcbac 

B acbcd 

C dcabc 

D cdabc 

Answer: A 

 

Q.4 Select the figure that will come next in the following figure series. 

Answer: D 

 

Q.5 In a code language, PERMUTATION is written as IBKFRMXMFLG. How will PUBLICSECTOR be written as in that language? 

A IRVEFVLBVMLK 

B ICUFEVLBVNLK 

C ICUFEVLBVMLJ 

D IRUEFVLBVMLK 

Answer: D 

 

Q.6 Which number will replace the question mark (?) in the following series? 5, 9, 18, 43, 92, 213, 382, ? 

A 328 

B 382 

C 617 

D 671 

Answer: D 

 

Q.7 How many triangles are present in the following figure ? 

A 14 

B 12 

C 13 

D 10 

Answer: C 

 

Q.8 Three of the following four letter-clusters are alike in a certain way and one is different. Pick the odd one out . 

A LOPK 

B MNQJ 

C CXGT 

D GSVE 

Answer: D 

 

Q.9 Select the set in which the numbers are related in the same way as are the numbers of the following set. (5, 2, 30) 

A (4, 3, 28) 

B (5, 1, 29) 

C (9, 2, 87) 

D (7, 8, 114) 

Answer: D 

 

Q.10 A paper is folded and cut as shown in the following figures. How will it appear when unfolded ? 

Answer: D

 

Q.11 Select the set in which the numbers are related in the same way as are the numbers of the following set. (4, 10, 23) 

A (7, 23, 49) 

B (4, 12, 21) 

C (2, 6, 15) 

D (3, 7, 17) 

Answer: C 

 

Q.12 Select the Venn diagram that best illustrates the relationship between the following classes. Raw material, Labourers, Machines 

Answer: B 

 

Q.13 Select the option that is related to the third letter-cluster in the same way as the second letter-cluster is related to the first letter-cluster. MARKET : IEONCV :: THERMO : ________ 

A JKCTKQ 

B JLZUKQ 

C PLDSKQ 

D PLBUKQ 

Answer: D

 

Q.14 Select the word-pair in which the two words are related in the same way as are the two words in the following word-pair. Spoke : Wheel 

A Printer : Computer 

B Word : Sentence 

C Tree : Branch 

D Square : Side 

Answer: B 

 

Q.15 Select the option that is related to the third number in the same way as the second number is related to the first number. 54 : 41 :: 32 : ____ 

A 17 

B 13 

C 11 

D 29 

Answer: B 

 

Q.16 In a family of 5 members, X is the sister of Y. M has two children and he is the son of E, who is the father-in-law of H. H has only one son. Y is not the grand-daughter of E. How is X related to E? 

A Grandson 

B Daughter 

C Sister 

D Grand-daughter 

Answer: D 

 

Q.17 Three of the following four words are alike in a certain way and one is different. Pick the odd word out. 

A Purse 

B Suitcase 

C Bag 

D Carpet 

Answer: D 

 

Q.18 Rahul has ₹340 in the denominations of ₹2 notes, ₹5 notes and ₹10 notes. The number of notes of each denomination is equal. What is the total number of notes that Rahul has? 

A 80 

B 20 

C 40 

D 60 

Answer: D 

 

Q.19 Which two signs should be interchanged in the following equation to make it correct?

24 ÷ 2 + 13 − 54 × 2 = 34

A − and ×

B + and ×

C − and ÷

D × and ÷

Answer: D 

 

Q.20 Arrange the following words in a logical and meaningful order. 1. Promotion 2. Application 3. Job appointment 4. Written test 5. Merit list 

A 2, 4, 5, 3, 1 

B 2, 5, 4, 3, 1 

C 4, 2, 5, 3, 1 

D 3, 2, 4, 1, 5 

Answer: A 

 

Q.21 If MASTER is coded as 26138402536 and GOVIND is coded as 142254481288, then how will BACKSPACE be coded as? 

A 4172393337125 

B 5172339317125 

C 5172933373215 

D 4162238321625 

Answer: D 

 

Q.22 Select the correct mirror image of the given figure when the mirror is placed to the right of the figure. 

Answer: A

 

Q.23 Three of the following four numbers are alike in a certain way and one is different. Pick the number that is different from the rest. 

A 9368 

B 4147 

C 7298 

D 8205 

Answer: C 

 

Q.24 Three statements are given, followed by three conclusions numbered I, II and III. Assuming the statements to be true, even if they seem to be at variance with commonly known facts, decide which of the conclusions logically follow(s) from the statements.

Statements: Some essays are poems. Some poems are directors. All directors are singers.

Conclusions: I. Some directors are poems. II. Some singers are essays. III. Some singers are poems. 

A Only conclusions I and III follow. 

B Only conclusions I and II follow. 

C Only conclusions II and III follow. 

D Only conclusion I follows. 

Answer: A 

 

Q.25 ‘Cardiologist’ is related to ‘Heart’ in the same way as ‘Chiropractor’ is related to ‘_________’. 

A Foot 

B Skin 

C Chest 

D Joints 

Answer: D 

 

General knowledge 

Instructions For the following questions answer them individually 

Q.26 Which of the following metals has an ore named Galena? 

A Copper 

B Nickel 

C Iron 

D Lead 

Answer: D 

 

Q.27 Which of the following terms refers to the running down or payment of a loan in instalments? 

A Amortisation 

B Backwardation 

C Credit creation 

D Discounted cashflow 

Answer: A 

 

Q.28 In June 2019, PepsiCo India proposed to invest $70 million to build a food manufacturing plant in: 

A Uttar Pradesh 

B Gujarat 

C Madhya Pradesh 

D Maharashtra 

Answer: A 

 

Q.29 During the passage of the Royal Titles Act 1876, the office of the British Prime Minister was occupied by: 

A William Ewart Gladstone 

B Benjamin Disraeli 

C Arthur Balfour 

D John Russell 

Answer: B 


Q.30 Which of the following states shares the longest boundary with China? 

A Sikkim 

B Himachal Pradesh 

C Arunachal Pradesh 

D Uttarakhand 

Answer: C 

 

Q.31 A region of computer memory where frequently accessed data can be stored for rapid access is called: 

A Cache 

B Cookie 

C Token 

D Plug-in 

Answer: A 

 

Q.32 Which of the following states separates Nepal from Bhutan? 

A Bihar 

B Sikkim 

C Assam 

D Odisha 

Answer: B 

 

Q.33 Who among the following was the first Sayyid ruler of Delhi? 

A Alam Shah 

B Muḥammad Shah 

C Mubārak Shah 

D Khizr Khan 

Answer: D 

 

Q.34 Rani-ki-Vav (the Queen’s Stepwell), which is in the UNESCO World Heritage List, is located in: 

A Madhya Pradesh 

B Gujarat 

C Rajasthan 

D Uttar Pradesh 

Answer: B 

 

Q.35 Who is the director of the film ‘PM Narendra Modi’? 

A Tigmanshu Dhulia 

B Hansal Mehta 

C Rakeysh Omprakash Mehra 

D Omung Kumar 

Answer: D 

 

Q.36 In which year was Google incorporated as a private company? 

A 2000 

B 1998 

C 2002 

D 2005 

Answer: B 

 

Q.37 Paan Singh Tomar, who was a seven-time national champion, was associated with which of the following sports? 

A Hockey 

B Swimming 

C Steeplechase 

D Shot Put 

Answer: C 

 

Q.38 The joint process of vapourisation and condensation is called: 

A Sublimation 

B Crystallisation 

C Chromatography 

D Distillation 

Answer: D 

 

Q.39 Who was the captain of the Indian women’s Kabaddi team that won silver at the 2018 Asian Games? 

A Payel Chowdhury 

B Shalini Pathak 

C Manpreet Kaur 

D Ritu Negi 

Answer: A 

 

Q.40 In the new Union cabinet, the portfolio of Micro, Small and Medium Enterprises was allocated to: 

A Sadananda Gowda 

B Ramvilas Paswan 

C Nitin Jairam Gadkari 

D Narendra Singh Tomar 

Answer: C 

 

Q.41 Which of the following are the highest-frequency electromagnetic waves? 

A Gamma Rays 

B Ultraviolet Rays 

C Microwaves 

D Radio Waves 

Answer: A 

 

Q.42 Who among the following is the 2019 winner of the $100,000 Nine Dots Prize? 

A Shashi Tharoor 

B Chetan Bhagat 

C Annie Zaidi 

D Arundhati Roy 

Answer: C 

 

Q.43 Panchavati, a key part of the Valmiki Ramayana is located in which state of India? 

A Maharashtra 

B Tamil Nadu 

C Uttar Pradesh 

D Uttarakhand 

Answer: A 

 

Q.44 What is the full form of GSTIN in relation to GST? 

A Goods and Services Tax Identification Note 

B Goods and Services Tax Identification Number 

C Goods and Services Tax Information Number 

D Goods and Services Taxation Income Number 

Answer: B 

 

Q.45 Which of the following political parties was NOT an ally of the BJP-led NDA in 2019 Lok Sabha elections? 

A DMK 

B Asom Gana Parishad 

C AIADMK 

D Shiromani Akali Dal 

Answer: A 

 

Q.46 In May 2019, the Government of India approved a new scheme which assures minimum monthly pension to all shopkeepers, retail traders and self employed persons after attaining the age of 60 years. How much is the pension amount? 

A ₹7,000 

B ₹10,000 

C ₹5,000 

D ₹3,000 

Answer: D 

 

Q.47 Who among the following is the author of ‘Kamayani’, the epic poem that is considered as one of the greatest literary works in Hindi? 

A Jaishankar Prasad 

B Premchand 

C Mohan Rakesh 

D Ramdhari Singh Dinkar 

Answer: A 

 

Q.48 On the occasion of ‘World No Tobacco Day’ on 31 May, which state banned e cigarettes? 

A Gujarat 

B Uttar Pradesh 

C Rajasthan 

D Assam 

Answer: C 

 

Q.49 Who among the following won a silver medal in the equestrian event at the 2018 Asian Games? 

A Fouaad Mirza 

B Ashish Malik 

C Jitender Singh 

D Rakesh Kumar 

Answer: A 

 

Q.50 The Bhupen Hazarika Setu, also called Dhola-Sadiya Bridge which connects Assam and _________. 

A Arunachal Pradesh 

B Sikkim 

C Meghalaya 

D West Bengal 

Answer: A 

 

Quantitative Aptitude 

Instructions For the following questions answer them individually 

Q.51 The given Bar Graph presents the Target and Actual production of AC Machines (numbers in thousands) of a factory over five months. 

The actual production of AC Machines in April was what percentage more than the average target production of AC Machines over five months ? 

A 11⅑% 

B 10⅑%
C 9% 

D 10% 

Answer: A 

 

Q.52 If cotθ = 1/√3, then the value of (2−sin2 θ)/ (1−cos2 θ) + (cosec2 θ + secθ) is: 

A 4 

B 7 

C 6 

D 5 

Answer: D 

 

Q.53 A is 20% more than B, B is 25% more than C, C is 60% less than D and D is 20% more than E. Based on the above information, which of the following is true? 

A C is 24% less than A. 

B E is 28% more than A. 

C A is 40% less than D. 

D D is 60% less than B. 

Answer: C 

 

Q.54 Two circles of radii 15 cm and 12 cm intersect each other, and the length of their common chord is 18 cm. What is the distance (in cm) between their centres ? 

A 12 + 3√7 

B 15 + √7 

C 12 + 2√7 

D 18 + √7 

Answer: A 

 

Q.55 The ratio of the incomes of A and is 2 : 3 and that of their expenditures is 1 : 2. If 90% of B’s expenditure is equal to the income of A, then what is the ratio of the savings of A and B? 

A 8:7 

B 1:1 

C 9:8 

D 3:2 

Answer: A 

 

Q.56 Two concentric circles are of radii 15 cm and 9 cm. What is the length of the chord of the larger circle which is tangent to the smaller circle? 

A 18 cm 

B 24 cm 

C 25 cm 

D 20 cm 

Answer: B 

 

Q.57 [(sec θ+tanθ)(1−sinθ)]/[cosec θ(1+cosθ)(cosec θ−cotθ)] is equal to: 

A secθ 

B cosecθ 

C sinθ 

D cosθ 

Answer: D 

Q.58 If 3√3 x3 − 2√2 y3 = (√3 x −√2 y)(Ax2Bxy + Cy2 ), then the value of (A2B2 + C2)is: 

A 7 

B 10 

C 1 

D 17 

Answer: A 

 

Q.59 Two trains of same length are running on parallel tracks in the same direction at 54 km/h and 42 km/h respectively. The faster train passes the other train in 63 seconds. What is the length (in metres) of each train? 

A 90 

B 81 

C 105 

D 210 

Answer: C 

 

Q.60 The value of [3÷ { 5−5÷(6−7)×8+9 }]/ (4+4×4÷4 of 4) is: 

A ⅓

B ⅟₄₅

C ⅟₉₀

D ⅟₁₈

Answer: C

 

Q.61 The volume of a right circular cone is 924 cm . If its height is 18 cm, then the area of its base (in cm ) is: 

A 154 

B 198 

C 176 

D 132 

Answer: A 

 

Q.62 The given Bar Graph presents the Target and Actual production of AC Machines (numbers in thousands) of a factory over five months. 

In which month the actual production of AC Machines was 25% more than the target production? 

A February 

B January 

C May 

D March 

Answer: D 

 

Q.63 In △ABC, AB = 7 cm, BC = 24 cm and AC = 25 cm.If is the centroid of the triangle, then what is the length (in cm) of BG ? 

A 8⅔

B 8⅓

C 9 

D 10 

Answer: B

 

Q.64 The given Bar Graph presents the Target and Actual production of AC Machines (numbers in thousands) of a factory over five months. 

The total target production of AC Machines in February, April and May was what percentage less than the total actual production of AC Machines overall the five months(correct to one decimal place)? 

A 47.6% 

B 46.2% 

C 47.1% 

D 46.8% 

Answer: A 

 

Q.65 If 30 persons take 10 days to complete a certain work working 8 hours a day, then 40 persons should work how many hours a day so that the work is completed in 6 days? 

A 8 

B 6

C 12 

D 10 

Answer: D 

 

Q.66 The simple interest on a certain sum for 3½ years at 10% per annum is ₹2,940. What will be the compound interest on the same sum for 2½ years at the same rate when interest is compounded yearly (nearest to a rupee)? 

A ₹2,227 

B ₹2,327 

C ₹2,372 

D ₹2,272 

Answer: D 

 

Q.67 There are 90 students in a class, out of which 70% are from village A and others are from village B. The average score of students from village B in a test is 20% more than that from village A. If the average score of all the students is 53, then what is the average score of the students from village B? 

A 60 

B 54 

C 64 

D 50 

Answer: A 

 

Q.68 If a + b + c = 5,a2 + b2 + c2  = 27, and a3 + b3 + c3 = 125, then the value of 4abc is: 

A -20 

B 20 

C 15 

D -15 

Answer: A 

 

Q.69 If cos2 θ − sin2 θ − 3cosθ + 2 = 0, 0 < θ < 90, then what is the value of 4cosecθ + cotθ

A 4 

B 3 

C 3√3 

D 4√3 

Answer: C 

 

Q.70 After allowing a discount of 10% on the marked price of an article,it is sold for ₹360. Had the discount not been given, the profit would have been 25%. What is the cost price of the article? 

A ₹360 

B ₹320 

C ₹350 

D ₹325 

Answer: B 

 

Q.71 If a2 + 4b2 + 49c2 + 18 = 2(2b + 28c a) , then the value of (3a + 2b + 7c) is: 

A 2 

B 0 

C 3 

D 1 

Answer: A 

 

Q.72 If the eight-digit number 342x18y6 is divisible by 72, then what is the value of √(9x + y) , for the largest value of y ? 

A 8 

B 2√7 

C 4√7 

D 6 

Answer: D

 

Q.73 In △ABC, D and are the points on sides AB and AC, respectively, such that DE ∥ BC. If DE : BC is 3 : 5, then (Area of △ADE) : (Area of quadrilateral DECB) is: 

A 3 : 4 

B 9 : 16 

C 5 : 8 

D 9 : 25 

Answer: B 

 

Q.74 The given Bar Graph presents the Target and Actual production of AC Machines (numbers in thousands) of a factory over five months. 

The ratio of the combined target production of AC Machines in January and April to that of the combined actual production of AC Machines in March and April was: 

A 5 : 6 

B 2 : 3 

C 3 : 2 

D 4 : 5 

Answer: A 

 

Q.75 A person sold an article at a loss of 8%. Had he sold it at a gain of 10.5%, he would have received ₹37 more. What is the cost price of the article? 

A ₹240 

B ₹250 

C ₹210 

D ₹200 

Answer: D 

 

English 

Instructions For the following questions answer them individually 

Q.76 Select the most appropriate word to fill in the blank. People’s will and determination can help them ______ most of the challenges of life. 

A overvalue 

B overview 

C overdo 

D overcome 

Answer: D 

 

Instructions In the sentence identify the segment which contains the grammatical error. 

Q.77 The Public Works Department has propose to construct an elevated corridor which will run parallel to the National highway. 

A to the National highway 

B which will run parallel 

C to construct an elevated corridor 

D The Public Works Department has propose 

Answer: D 

 

Q.78 I visited my friend to whom I had made an appointment. 

A to whom 

B I had made 

C an appointment 

D I visited my friend 

Answer: A 

 

Instructions For the following questions answer them individually 

Q.79 Select the word which means the same as the group of words given. One who does not tire easily 

A indelible 

B inevitable 

C indefatigable 

D infallible 

Answer: C 

 

Q.80 Select the most appropriate meaning of the given idiom. throw in the towel 

A face the situation 

B drop something 

C think of a solution 

D admit defeat 

Answer: D 

 

Q.81 Select the word which means the same as the group of words given. A group of singers in a church 

A host 

B choir 

C troop 

D band 

Answer: B 

 

Q.82 Given below are four jumbled sentences. Out of the given options pick the one that gives their correct order. 

A. He often asked questions which were strange and witty. 

B. Emperor Akbar was in the habit of putting riddles and puzzles to his courtiers. 

C. Once he asked a strange question that confused everyone. 

D. It took much wisdom to answer these questions. 

A CABD 

B BADC 

C BACD 

D ABCD 

Answer: B 

 

Instructions: In the following passage some words have been deleted. Fill in the blanks with the help of the alternatives given. Select the most appropriate option for each blank. 

Comprehension: Big Ben is the UK’s most iconic symbol. Since 2017, the Elizabeth Tower and the bell inside have been (1) ______ renovation. The structure is 160 years old and (2) ______ regular maintenance. (3) ______ many years, people had been painting the hands and numbers (4) ______ the clock black, but last Thursday, the workers (5) ______ the clock’s original colour, Prussian Blue! 

Q.83 Select the most appropriate option for blank No. 1. 

A undertaking 

B understanding 

C undergoing 

D undermining 

Answer: C 

 

Q.84 Select the most appropriate option for blank No. 2. 

A was needing 

B has needed 

C is needed 

D need 

Answer: B 

 

Q.85 Select the most appropriate option for blank No. 3. 

A About 

B For 

C In 

D Before 

Answer: B 

 

Q.86 Select the most appropriate option for blank No. 4. 

A at 

B for 

C of 

D to 

Answer: C 

 

Q.87 Select the most appropriate option for blank No. 5. 

A concealed 

B discovered 

C imparted 

D invented 

Answer: B 

 

Instructions For the following questions answer them individually 

Q.88 Given below are four jumbled sentences. Out of the given options pick the one that gives their correct order. 

A. Most of them are in jail because of their circumstances. 

B. Not all prisoners are hardcore criminals. 

C. So, they deserve a chance to rehabilitate themselves. 

D. Therefore, learning a trade would help them reintegrate with society. 

A BACD 

B ABCD 

C BDCA 

D DCBA 

Answer: A 

 

Q.89 Select the most appropriate antonym of the given word. SUCCINCT 

A lengthy 

B pithy 

C curt 

D terse 

Answer: A 

 

Q.90 Select the most appropriate synonym of the given word. HIND 

A front 

B near 

C first 

D rear 

Answer: D 

 

Q.91 Select the most appropriate option to substitute the underlined segment in the given sentence. If there is no need to substitute it, select No improvement. I hope never to have another so experience as I had in Puri during Cyclone Fani. 

A as experience as 

B such experience as 

C No improvement 

D same experience as 

Answer: B 

 

Q.92 Select the most appropriate word to fill in the blank. In spite of being born in an ______ family, he chose to fight all odds and emerged as a not able statesman. 

A intelligent 

B immaculate 

C impeccable 

D impoverished 

Answer: D 

 

Q.93 Select the wrongly spelt word. 

A assure 

B assuredly 

C assurable 

D assurence 

Answer: D 

 

Q.94 Select the most appropriate option to substitute the underlined segment in the given sentence. If there is no need to substitute it, select No improvement. Oh, what a lovely necklace! You need no buy such an expensive gift. 

A hadn’t needed to buy 

B No improvement 

C didn’t need to buying 

D needn’t have bought 

Answer: D 

 

Q.95 Select the wrongly spelt word. 

A barren 

B berrel 

C barrier 

D berry 

Answer: B 

 

Q.96 Select the correct passive form of the given sentence. The gardener has mowed the lawn. 

A The lawn has been mowed by the gardener. 

B The lawn is mowed by the gardener. 

C The gardener has been mowed by the lawn. 

D The lawn was mowed by the gardener. 

Answer: A 

 

Q.97 Select the most appropriate antonym of the given word. EXOTIC 

A colourful 

B ordinary 

C alien 

D curious 

Answer: B 

 

Q.98 Select the most appropriate synonym of the given word. INNUENDO 

A verification 

B proof 

C implication 

D evidence 

Answer: C 

 

Q.99 Select the most appropriate meaning of the given idiom. to air dirty linen in public 

A to hang out clothes in the open 

B to stand up and fight 

C to continue to complain 

D to discuss private affairs in public 

Answer: D 

 

Q.100 Select the correct indirect form of the given sentence. Mrs. Gupta said to me, “Why are these boys standing in the sun?” 

A Mrs. Gupta asked me why were these boys standing in the sun. 

B Mrs. Gupta said to me that why are these boys standing in the sun. 

C Mrs. Gupta asked me why those boys were standing in the sun. 

D Mrs. Gupta told me why those boys are standing in the sun. 

Answer: C 

SSC CHSL Tier-I 3 July 2019 Shift-III Previous Year Paper

SSC CHSL

(3 July 2019 Shift-III) 

Reasoning 

Instructions For the following questions answer them individually 

Q.1 Select the figure that will come next in the following figure series. 

Answer: B 

 

Q.2 The total age of a mother and her son is 60 years. The difference between their ages is 30 years. Find out the age of the mother. 

A 40 years 

B 45 years 

C 35 years 

D 50 years 

Answer: B 

 

Q.3 Three of the following four words are alike in a certain way and one is different. Pick the odd word out. 

A Bark 

B Croak 

C Larva 

D Bleat 

Answer: C 

 

Q.4 Select the word-pair in which the two words are related in the same way as are the two words in the following word-pair. Fan : Electricity 

A Petrol : Fuel 

B Bike : Kick 

C Vehicles : Diesel 

D Cooler : Water 

Answer: C 

 

Q.5 Select the option in which the given figure (X) is embedded(rotation not allowed). 

Answer: D 

 

Q.6 Two statements are given, followed by three conclusions numbered I, II and III. Assuming the statements to be true, even if they seem to be at variance with commonly known facts, decide which of the conclusions logically follow(s) from the statements. Statements: All breads are cakes. All eggs are cakes. 

Conclusions: I. Some cakes are breads. II. Some breads are eggs. III. No bread is an egg. 

A All the conclusions follow. 

B Conclusion I and either conclusion II or III follow. 

C Only conclusions I and III follow. 

D Only conclusions I and II follow. 

Answer: B 

 

Q.7 Select the option that depicts how the given transparent sheet of paper would appear if it is folded at the dotted line. 

Answer: A 

 

Q.8 Select the number-pair in which the two numbers are related in the same way as are the two numbers of the following number-pair. 7 : 729 

A 6 : 343 

B 4 : 64 

C 8 : 1000 

D 5 : 125 

Answer: C 

 

Q.9 There are fourteen teams playing in a tournament. If every team plays one match with every other team, how many matches will be played in the tournament ? 

A 101 

B 91 

C 66 

D 78 

Answer: B 

 

Q.10 If MAGENTA is coded as 1317514201, then how will VISCOUS be coded as ? 

A 229193152119 

B 228193142118 

C 239203152219 

D 218193162018 

Answer: A

 

Q.11 Select the option that is related to the third letter-cluster in the same way as the second letter-cluster is related to the first letter-cluster. CQHL : FTEI :: OPAR : ? 

A RSXO 

B RSXP 

C YTXO 

D RTYO 

Answer: A 

 

Q.12 Select the letter cluster that will come next in the following series. DOP, FPN, HQL, JRJ, ? 

A KSH 

B LSI 

C LSH 

D KTI 

Answer: C 

 

Q.13 How many triangles are present in the following figure ? 

A 29 

B 27 

C 31 

D 30 

Answer: A 

 

Q.14 Select the combination of letters that when sequentially placed in the gaps of the given letter series will complete the series. H _ _ E P C K _ _ C D E P _ K F H C _ E P C _ F 

A DCFHFDD 

B CDHHDCK 

C CCFHKDK 

D CDFHCDK 

Answer: D 

 

Q.15 Select the Venn diagram that best illustrates the relationship between the following classes. Women, Sisters, Mothers 

Answer: C 

 

Q.16 Three of the following four letter-clusters are alike in a certain way and one is different. Pick the odd one out. 

A TUWZ 

B BCEH 

C LMPS 

D KLNQ 

Answer: C 

 

Q.17 Two different positions of the same dice are shown. How many dots will be on the top if four dots are at the bottom ? 

A 2 

B 1 

C 3 

D 5 

Answer: C 

 

Q.18 ‘ A + B ’ means ‘A is the brother of B’. 

A B ’ means ‘A is the husband of B’. 

A × B ’ means ‘A is the mother of B’. 

A ÷ B ’ means ‘A is the sister of B’. 

If P + R ÷ T K × O × C , then how is P related to O ? 

A Brother 

B Paternal Grandfather 

C Father 

D Paternal Uncle 

Answer: D 

 

Q.19 In a code language, ELASTIC is written as DOHVFLW. How will SMARTER be written as in that language? 

A DOTUUHX 

B DPVTUGW 

C DPVUUHW 

D EQVUUHW 

Answer: C 

 

Q.20 Which two signs should be interchanged to make the given equation correct? 

20 + 5 × 3 ÷ 3 − 1 = 14 

A ÷ and

B × and

C × and − 

D ÷ and × 

Answer: A 

 

Q.21 Three of the following four numbers are alike in a certain way and one is different. Pick the number that is different from the rest. 

A 183 

B 179 

C 173 

D 181 

Answer: A 

 

Q.22 In a code language, ELECTION is written as DKDBSHNM. How will EXAMPLE be written in the same language? 

A DYZLOKD 

B FWZLOKD 

C DWZLOKF 

D DWZLOKD 

Answer: D

 

Q.23 Arrange the following in a logical and meaningful order. 1. World Cup semi-final matches 2. World Cup finals match 3. Organising the World Cup 4. Finalising the participating teams 5. Winner of the World Cup 

A 5, 2, 3, 1, 4 

B 4, 1, 3, 2, 5 

C 4, 3, 1, 2, 5 

D 4, 1, 2, 3, 5 

Answer: C 

Q.24 ‘Glaucoma’ is related to ‘Eyes’ in the same way as ‘Arthritis’ is related to ‘_________’. 

A Ears 

B Joints 

C Liver 

D Arteries 

Answer: B 

 

Q.25 Select the correct mirror image of the given combination when the mirror is placed to the right. 

Answer: D 

 

General knowledge 

Instructions For the following questions answer them individually 

Q.26 India lifted South Asian Football Federation SAFF Women’s Championship for how many times in a row in March 2019 ? 

A 4th

B 6th

C 3rd

D 5th 

Answer: D 

 

Q.27 Who among the following invented the computer language, COBOL? 

A Guido van Rossum 

B Brendan Eich 

C Grace Murray Hopper 

D John McCarthy 

Answer: C 

 

Q.28 In the famous Rohilla War in the year 1774, the Rohillas were defeated by the Nawab of Awadh named: 

A Asif Jah Mirza 

B Shuja-ud-Daula 

C Asaf-ud-Daula 

D Yamin-ud-Daula 

Answer: B 

 

Q.29 The power to enquire into and decide all doubts and disputes arising out of election of the President is vested in: 

A The Supreme Court 

B The Prime Minister of India 

C The Election Commission 

D The Chairman of Rajya Sabha 

Answer: A 

 

Q.30 Second generation computers can be characterised largely by their use of ________. 

A Integrated circuits 

B Transistors 

C Microprocessors 

D Vacuum tubes 

Answer: B 

 

Q.31 As of May 2019, which player has scored the most international runs in the world in all the three formats (Test Match, ODI Match and T-20 Match) of cricket combined? 

A Virat Kohli 

B Ricky Ponting 

C Don Bradman 

D Sachin Tendulkar 

Answer: D 

 

Q.32 Which one of the following is a monetary principle stating that “bad money drives out good”? 

A Gresham’s law 

B Pareto efficiency 

C The multiplier effect 

D Marshall’s scissors analysis 

Answer: A 

 

Q.33 Which force helps swimmers float in water? 

A Muscular force 

B Magnetic force 

C Buoyant force 

D Frictional force 

Answer: C 

 

Q.34 What is the rank of India on Human Development Index 2018? 

A 130 

B 181 

C 21 

D 210 

Answer: A 

 

Q.35 On which river is Polavaram irrigation project being built? 

A Kaveri 

B Son 

C Tungabhadra 

D Godavari 

Answer: D 

 

Q.36 According to Census 2011, which state has the highest density of population? 

A Maharashtra 

B West Bengal 

C Bihar 

D Uttar Pradesh 

Answer: C 

 

Q.37 Oman author Joka Alharthi won the ‘2019 Man Booker International Prize’ for which of the following novels? 

A Milkman 

B The Years 

C Celestial Bodies 

D The Pine Islands 

Answer: C 

 

Q.38 As per the Constitution of India, Panchayats at the intermediate level may NOT be constituted in a State having a population not exceeding: 

A Forty lakhs 

B Twenty lakhs 

C Ten lakhs 

D Thirty lakhs 

Answer: B 

 

Q.39 _______ is a metamorphic rock form characterised by banding caused by segregation of different types of rock, typically light and dark silicates. 

A Quartzite 

B Slate 

C Gneiss 

D Marble 

Answer: C 

 

Q.40 Which of the following places is famous for Chikankari embroidery? 

A Indore 

B Chennai 

C Hyderabad 

D Lucknow 

Answer: D 

 

Q.41 Which of the following is the highest award (in the order of precedence) for military services in India? 

A Veer Chakra 

B Ashok Chakra 

C Param Vir Chakra 

D Kirti Chakra 

Answer: C 

 

Q.42 Which country won the ‘2019 Sultan Azlan Shah Cup Hockey’ title? 

A Australia 

B India 

C Malaysia 

D South Korea 

Answer: D 

 

Q.43 Which is the new state bird of Andhra Pradesh declared by the State Government in June 2018? 

A Black Crested Bulbul 

B Rose-ringed Parrot 

C Great Hornbill 

D Sarus Crane 

Answer: B 

 

Q.44 Deficiency of which of the following nutrients is the most common cause of goiters? 

A Vitamin C 

B Iron 

C Calcium 

D Iodine 

Answer: D 

 

Q.45 To which state does the traditional folk dance ‘Gotipua’ belong? 

A Chhattisgarh 

B West Bengal 

C Bihar 

D Odisha 

Answer: D 

 

Q.46 Kolleru Lake which is one of the largest freshwater lakes in India is located in which state? 

A Jammu & Kashmir 

B Manipur 

C Andhra Pradesh 

D Rajasthan 

Answer: C 

 

Q.47 Who was recently elected as the President of Indonesia for the second term? 

A Abdurrahman Wahid 

B Megawati Sukarnoputri 

C Joko Widodo 

D Susilo Bambang Yudhoyono 

Answer: C 

 

Q.48 Who among the following was awarded the third Sayaji Ratna Award in November 2018 which has been established in the memory of erstwhile Baroda ruler Sayajirao Gaekwad III ? 

A Shiv Nadar 

B Azim Premji 

C Mohanlal 

D Amitabh Bachchan 

Answer: D 

 

Q.49 Which day has been proclaimed as The International Day for Biological Diversity (IDB) to increase understanding and awareness of biodiversity issues by The United Nations? 

A 24 Apr 

B 18 Oct 

C 22 May 

D 10 November 

Answer: C 

 

Q.50 Which emperor’s army did Maharana Pratap fight in the Battle of Haldighati ? 

A Ibrahim Lodi 

B Mahmud Shah 

C Akbar 

D Humayun 

Answer: C 

 

Quantitative Aptitude 

Instructions For the following questions answer them individually 

Q.51 If a 10-digit number 46789x531y is divisible by 72, then the value of (2x + 5y), for the largest value of x is:

A 10 

B 38 

C 28 

D 16 

Answer: C 

 

Q.52 The compound interest on a certain sum for 3 years at 15% p.a., interest compound yearly, is ₹4167. What is the simple interest on the same sum 4⅘ in years at the same rate ? 

A ₹5760 

B ₹4800 

C ₹6144 

D ₹6000 

Answer: A 

Q.53 The value of is: 

A 1¾

B 2½

C 3½

D 4 

Answer: C 

 

Q.54 If x + y + z = 19,x2 + y2 + z2 = 133, then the value of  x3 + y3 + z3 − 3xyz is: 

A 361 

B 352 

C 380 

D 342 

Answer: A 

 

Q.55 In a class of 80 students, the ratio of the urban to the rural is 5 : 3. Ina test, the average score of the rural students is 40% more than that of the urban students. If the average score of all the students is 69, then what is the average score of the rural students ? 

A 76 

B 92 

C 80 

D 84 

Answer: D 

 

Q.56 In a circle with centre O, a diameter AB and a chord CD intersect each other at E, AC and AD are joined. If ∠ BOC= 48 and ∠ AOD = 100 , then what is the measure of ∠ CEB ? 

A 82 

B 74 

C 72 

D 78 

Answer: B 

 

Q.57 18 men can complete a work in 9 days. After they have worked for 5 days, 6 more men join them. How many days will they take to complete the remaining work ? 

A 2½

B 2 

C 3½ 

D 3 

Answer: D 

 

Q.58 A man gets a discount of 30% and then 20% on his food bill of ₹1,250. How much amount does he has to pay? 

A ₹350 

B ₹550 

C ₹500 

D ₹700 

Answer: D 

 

Q.59 The given Bar Graph presents Income and Expenditure (in crores of Rupees) of a company for the five years, 2014 to 2018. 

What is the ratio of total Expenditure to total Income of the company in 2014, 2016 and 2017 ? 

A 3:4 

B 5:6 

C 13:18 

D 15:16 

Answer: B 

 

Q.60 In a △ ABC,D and E are two points on sides AB and BC, respectively such that AD : DB = 2: 3 and DE ∥ AC. If the area of △ ADE is equal to 18 cm 2 , then what is the area (in cm?) of △ ABC ? 

A 54 

B 40.5 

C 75 

D 45 

Answer: C

 

Q.61 Let △ABC ∼ △QPR and ar(△ABC) /ar(△PQR) = 9/4. If AB = 12 cm, BC = 6 cm and AC = 9 cm,then QR is equal to: 

A 6 cm

B 9 cm 

C 12 cm 

D 15 cm 

Answer: A 

 

Q.62 In △ABC, AD is the bisector of ∠BAC, meeting BC at D. If AC = 21 cm, BC = 12 cm and the length of BD is 2 cm less than DC, then the length of side AB is: 

A 18 cm 

B 14 cm 

C 15 cm 

D 10 cm 

Answer: C 

 

Q.63 If 8(x + y)3− (x y3) = (x + 3y)(Ax2 + Bxy + Cy2 ), then the value of (A – B – C) is: 

A 14 

B -6 

C -2 

D 10 

Answer: C

 

Q.64 An article is sold for ₹360 after allowing discount of 20% on its marked price. Had the discount not been allowed, the profit would have been 50%. The cost price of the article is: 

A ₹300 

B ₹350 

C ₹320 

D ₹ 360 

Answer: A 

 

Q.65 If 9a2 +16b2 + c2 + 25 = 24(a + b), then the value of (3a + 4b + 5c) is: 

A 7 

B 10 

C 6 

D 9 

Answer: A 

 

Q.66 Two trains of equal length travelling in opposite directions at 72 km/h and 108 km/h cross each other in 10 seconds. In how much time(in seconds) does the first train cross a platform of length 350 m? 

A 30 

B 24 

C 32 

D 36 

Answer: A 

 

Q.67 An article is sold for ₹x. If it is sold at 33⅓% of this price, there is a loss of 20%. What is the percentage profit when it is sold for ₹x ? 

A 125 

B 120 

C 140 

D 130 

Answer: C 

 

Q.68 If tanx = cot(60 ∘ + 6x),then what is the value of x ? 

A 30/2

B 15/2

C 10

D 12

Answer: A 

 

Q.69 If (1/1+cosecθ 1/1−cosecθ ) cosθ = 2,0 < θ < 90, then the value of sin2 θ + cot2 θ + sec2θ is: 

A 3½

B 1

C 2 

D 2½

Answer: A 

 

Q.70 What is the simplified value of 5 ÷ 10 of 10 × 4 + 4 ÷ 4 of 4 × 10 − (10 − 4) ÷ 16 × 4 = ?

A 21 

B 58.5 

C 2.5 

D 1.2 

Answer: D 

 

Q.71 The given Bar Graph presents Income and Expenditure (in crores of Rupees) of a company for the five years, 2014 to 2018. 

In which year is the Expenditure more than 40% as compared to the Expenditure in the previous year? 

A 2015 

B 2016 

C 2017 

D 2018 

Answer: A 

 

Q.72 The price of an article increases by 20% every year. If the difference between the prices at the end of third and fourth years is ₹259.20, then 40% ofthe price (in ₹) at the end of second year is: 

A 432 

B 484 

C 472 

D 384 

Answer: A

 

Q.73 The length of a rectangular park is 20 m more than its breadth. If the cost of fencing the park at ₹53 per metre is ₹21200, then what is the area (in m2) of the park ? 

A 8925 

B 9240 

C 9504 

D 9900 

Answer: D 

 

Instructions: The given Bar Graph presents Income and Expenditure (in crores of Rupees) of a company for the five years, 2014 to 2018. 

Q.74 The total Income of the company in 2015, 2017 and 2018 is approximately what percent less than the total Expenditure in the five years ? 

A 22 

B 21 

C 26 

D 24 

Answer: C 

 

Q.75 The average Income(per year) of the company in five years is what percentage more than its Expenditure in 2015 ? 

A 20.8 

B 22.4 

C 24.2 

D 24.6 

Answer: B 

 

English 

Instructions For the following questions answer them individually 

Q.76 Select the most appropriate meaning of the given idiom. Mad as a hatter 

A very upset 

B old fashioned 

C eccentric 

D superstitious 

Answer: C 

 

Q.77 Select the word which means the same as the group of words given. The study of human history and prehistory through the excavation of sites 

A geology 

B anthropology 

C psephology 

D archaeology 

Answer: D 

 

Q.78 Select the most appropriate option to substitute the underlined segment in the given sentence. If there is no need to substitute it, select No improvement. The civic bodies in the capital have formed teams to crack down on coaching centres found violated a law. 

A found to violated a law 

B finding to violate the law 

C found violating the law 

D No improvement 

Answer: C 

 

Q.79 Select the most appropriate meaning of the given idiom. Going over one’s head 

A something one didn’t expect 

B unable to function as one used to 

C beyond one’s capacity to understand 

D unable to take a decision 

Answer: C 

 

Q.80 Select the wrongly spelt word. 

A resembel 

B require 

C reverse 

D reproduce 

Answer: A 

 

Q.81 Select the most appropriate word to fill in the blank. The prisoner held up his fist in a defiant ______ as he was led out of the courtroom. 

A wave 

B token 

C salute 

D gesture 

Answer: D 

 

Q.82 In the sentence identify the segment which contains the grammatical error. Mrs. Sangeeta Malik is one of the favourite teacher of most children in our school. 

A of most children 

B Mrs. Sangeeta Malik is 

C in our school 

D one of the favourite teacher 

Answer: D 

 

Q.83 Select the most appropriate antonym of the given word. RESERVED 

A modest 

B communicative 

C quiet 

D placid 

Answer: B 

 

Q.84 Select the most appropriate synonym of the given word. HANDY 

A hard 

B convenient 

C useless 

D clumsy 

Answer: B 

 

Q.85 Select the most appropriate synonym of the given word. THWART 

A aid 

B support 

C face 

D impede 

Answer: D 

 

Instructions: In the following passage some words have been deleted. Fill in the blanks with the help of the alternatives given. Select the most appropriate option for each blank. 

Comprehension: Every morning, nine-year old Khushi becomes guardian to her five-year old sister Ankita as (1) ______ hold hands and cross the railway tracks to (2) ______ the school. The sisters leave home (3) ______ 7 am with school bags larger (4) ______ their torsos. The 2 km route they walk on foot to reach school (5) ______ includes walking past a railway crossing and through the lanes of an industrial area used by mini trucks and carrier vehicles. 

Q.86 Select the most appropriate option for blank No. 1. 

A them 

B she 

C they 

D we 

Answer: C 

 

Q.87 Select the most appropriate option for blank No. 2. 

A come 

B reach 

C go 

D arrive 

Answer: B 

 

Q.88 Select the most appropriate option for blank No. 3. 

A from 

B on 

C at 

D in 

Answer: C 

 

Q.89 Select the most appropriate option for blank No. 4. 

A with 

B than 

C to 

D from 

Answer: B 

 

Q.90 Select the most appropriate option for blank No. 5. 

A evenly 

B daily 

C hardly 

D widely 

Answer: B 

 

Instructions For the following questions answer them individually 

Q.91 Select the most appropriate option to substitute the underlined segment in the given sentence. If there is no need to substitute it, select No improvement. The Urban Art Commission has suggested that the landscape be developing by adding water fountains around the sculpture. 

A to be developed by 

B No improvement 

C be developed by 

D be developed with 

Answer: C 

 

Q.92 Select the word which means the same as the group of words given. An instrument used for measuring atmospheric pressure 

A voltmeter 

B thermometer 

C barometer 

D altimeter 

Answer: C 

 

Q.93 Select the wrongly spelt word. 

A nerve 

B neglect 

C nerveous 

D negative 

Answer: C 

 

Q.94 Select the most appropriate antonym of the given word. INSTANT 

A immediate 

B current 

C delayed 

D quick 

Answer: C 

 

Q.95 Given below are four jumbled sentences. Out of the given options pick the one that gives their correct order. 

A. At this speed the rocket soon attained the height of 190 miles above the earth. 

B. At 9 am the great rocket lifted in the air with a mighty roar. 

C. It rose smoothly at first, then quickened to a speed of 17,500 miles an hour. 

D. Gagarin now pulled himself towards the window to look down. 

A ADBC 

B BADC 

C CBAD 

D BCAD 

Answer: D 

 

Q.96 Select the correct passive form of the given sentence. The new gardener looks after the plants well. 

A The plants are looked after well by the new gardener. 

B The plants were looked after well by the new gardener. 

C The new gardener is looked after well by the plants. 

D The plants looked after well by the new gardener. 

Answer: A 

 

Q.97 Select the correct indirect form of the given sentence. Father said to me, “Are you going to keep me waiting all night?’ 

A Father told me that you are going to keep him waiting all night. 

B Father asked me if you were going to keep him waiting all night. 

C Father asked me are you going to keep me waiting all night. 

D Father asked me if I was going to keep him waiting all night. 

Answer: D 

 

Q.98 Given below are four jumbled sentences. Out of the given options pick the one that gives their correct order. 

A. The subjects taught in the Gurukul varied from Sanskrit to Mathematics. 

B. Indian education has its roots in the ancient ages. 

C. However this system changed when the British came to India. 

D. In those days the Gurukul system was followed. 

A BDAC 

B DCAB 

C CBAD 

D BCAD 

Answer: A 

 

Q.99 Select the most appropriate word to fill in the blank. In view of his transfer to a far off place, he had to ______ his promotion. 

A claim 

B forgo 

C resign 

D assert 

Answer: B 

 

Q.100 In the sentence identify the segment which contains the grammatical error. I expect to return from Singapore during about a week’s time. 

A from Singapore 

B during about 

C a week’s time 

D I expect to return 

Answer: B 

×

Hello!

Click one of our representatives below to chat on WhatsApp or send us an email to info@vidhyarthidarpan.com

×